227532203-Nbme-15

March 26, 2018 | Author: sharrisanyae | Category: Hemoglobin, Lymphocyte, T Cell, Lung, Respiratory System


Comments



Description

NBme 151) A 50-year-old man comes to the physician because of a 2-week history of progressive shortness of breath while climbing stairs to his office every morning. He reports no other problems, but he is concerned because his father had a major myocardial infarction at the age of 52 years. His pulse is 110/min and regular, respirations are 16/min, and blood pressure is 135/95 mm Hg. The lungs are clear to auscultation. Cardiac examination shows normal heart sounds with a physiologic split of S2. Stress echocardiography shows hypokinesis of the posterior left ventricle with increasing activity levels. Which of the following is the most likely cause of the posterior left ventricular findings in this patient? Disruption of the sympathetic nerves to the left ventricle Extravascular compression of the coronary arteries Increased left ventricular end-diastolic pressure Increased myocardial oxygen consumption Stenosis of the right coronary artery 2) A 75-year-old woman comes to the physician because of a 3-month history of an enlarging lesion on her face. Physical examination shows a 1.5-cm, brown-black, mottled, scaly lesion with irregular borders. Microscopic examination of a biopsy specimen of the lesion shows atypical melanocytes spread along the basilar layer of the epidermis. Which of the following is the most likely cause of these findings? Acanthosis nigricans Actinic keratosis Compound nevus Lentigo maligna Seborrheic keratosis 3) A 13-year-old boy is scheduled to receive chemotherapy for a leukemia that has the histologic features of malignant lymphocytes. This neoplasm is further typed for cell surface and intracellular markers specific for lymphocyte subsets. The neoplastic cells do not express the following markers: CD4, CD8, surface IgM, surface IgG, cytoplasmic IgM and μ-heavy chain, cytoplasmic IgG, and γ- heavy chain. The leukemic cells express class I MHC molecules and show rearrangement of the T- lymphocyte receptor β-chain gene D and J segments. Which of the following is the normal counterpart of these malignant lymphocytes? Activated cytolytic effector T lymphocytes in the circulation Mature IgM-secreting B lymphocytes in the lymph node Mature immunoglobulin-secreting plasma cells in the lymph node Pre-B lymphocyte progenitor of mature B lymphocytes in the bone marrow T-lymphocyte thymocytes localized to the thymic cortex 4 ) A 70-year-old man has a 4-month history of weight loss, abdominal pain, and diarrhea. Stool analysis shows increased excretion of neutral fat and muscle fiber. A d-xylose test for carbohydrate absorption shows no abnormalities. Examination of tissue obtained on intestinal biopsy shows no abnormalities. This patient is most likely to respond favorably to administration of which of the following agents? Antibiotics Azathioprine Intrinsic factor Pancreatic enzymes Prednisone 5) A 20-year-old woman comes to the physician because her menstrual period is 1 week late. Menses had occurred at regular 28-day intervals. She had unprotected sexual intercourse with her boyfriend 20 days ago. A urine pregnancy test result is positive. Which of the following best describes the stage of development of the embryo at this time? The cytotrophoblast is present, but the syncytiotrophoblast has not yet formed Gastrulation is complete, but there are only two germ layers The neural plate is present, but the neural tube is not yet complete Placental development is complete, but the embryo is resistant to teratogens The sclerotome cells have begun migrating, but the somites have not yet formed 6) A 5-year-old boy is stung on his foot by a bee. Within 30 minutes the local area is edematous. The extravascular accumulation of fluid is most directly related to which of the following? Demargination of leukocytes Fibrin thrombi Gap formation between endothelial cells Vasoconstriction Vasodilation 7) The vaccine against diphtheria confers protective immunity by inducing the formation of antibodies against a preparation composed of which of the following? Killed bacterial cells Live attenuated bacterial cells Purified bacterial peptidoglycan Purified capsular polysaccharide Purified inactivated toxin 8) A 2-month-old girl is brought to the physician for a follow-up examination 2 days after a complete blood count was found to be abnormal. Her blood group is A, Rh-positive. She was born at term to a healthy 23-year-old woman whose blood group is O, Rh-positive. Birth weight was 3890 g (8 lb 9 oz). Pregnancy and delivery were uncomplicated. The patient has been active and feeding well. Her temperature is 37.6°C (99.7°F), pulse is 120/min, and respirations are 30/min. Physical examination shows no abnormalities. Laboratory studies done since birth are shown. Patient Age 24 Hours 1 Week 1 Month Hemoglobin (g/dL) (N=11−15) 16.9 15.7 12.7 Hematocrit (%) (N=28−45) 51 49 39 Leukocyte count (/mm3) (N=5000−19,500) 17,200 15,600 12,900 Segmented neutrophils 4% 3% 5% Eosinophils 4% 4% 2% Basophils 4% 2% 1% Lymphocytes 69% 73% 77% Monocytes 19% 18% 15% Platelet count (/mm3) (N=150,000−400,000) 185,000 198,000 205,000 Which of the following is the most likely diagnosis? Alloimmune hemolytic disease of the newborn Congenital cytomegalovirus infection Congenital neutropenia DiGeorge syndrome Severe combined immunodeficiency 9) A 50-year-old man comes to the emergency department because of a 2-week history of progressive shortness of breath. His pulse is 90/min, respirations are 26/min, and blood pressure is 120/80 mm Hg. Physical examination shows no other abnormalities. Laboratory studies show: Arterial Pco2 30 mm Hg Arterial Po2 96 mm Hg Arterial O2 content 12 vol% (N=17%–21%) Mixed venous Po2 36 mm Hg Mixed venous O2 content 8 vol% (N=10%–16%) Which of the following is the most likely explanation for these findings? Anemia Drug-induced alveolar hypoventilation Residence at a high altitude Severe regional mismatching of alveolar ventilation and pulmonary capillary perfusion Voluntary hyperventilation 10) An 18-year-old man is brought to the physician by his father for a sports physical examination. He is a defensive end on his high school's football team and a champion wrestler. After his father leaves the examining room, the patient tells the physician that he is embarrassed because of enlarged breasts; he is worried that his classmates will make fun of him. He is extremely muscular, and there is no evidence of obesity. He is 173 cm (5 ft 8 in) tall and weighs 86 kg (190 lb); BMI is 29 kg/m2. His sexual development is Tanner stage 5. Physical examination shows 2-cm soft mounds of tissue beneath the areola bilaterally. Which of the following is the most appropriate next step by the physician? Reassurance Inquiry about a history of anabolic steroid use Measurement of serum testosterone concentration Mammography to rule out breast cancer Referral for a breast reduction operation 11) A study is conducted of patients with restrictive airway disease who have BMIs above normal. Results show a significant correlation (r = –0.25) between body fat and lung capacity. Which of the following is the most accurate conclusion based on these data? Increased body fat is associated with higher lung capacity Increased body fat is associated with lower lung capacity Twenty-five percent of individuals with increased body fat have a higher lung capacity Twenty-five percent of individuals with increased body fat have a lower lung capacity No relation exists between body fat and lung capacity 12) A 6-year-old boy is brought to the physician by his mother because of a 3-month history of pubic and axillary hair growth. On previous visits, he has been at the 50th percentile for height and weight. Pubic and axillary hair development is Tanner stage 2. Physical examination shows mild enlargement of the testes. If left untreated, which of the following combinations would best describe the likely course of growth in this patient? Height Percentile 1 Year From Now As an Adult 25th 25th 25th 75th 50th 25th 50th 75th 75th 25th 75th 75th 13) A 45-year-old woman (III,7) comes to the physician for a routine health maintenance examination. Several people in her family, including her son (IV,3), have hereditary telangiectasia. A pedigree is shown. Physical examination and full medical evaluation show no abnormalities. Which of the following best explains the reason for this patient's health? Gonadal mosaicism Incomplete penetrance Nonpaternity Somatic mosaicism Variable expressivity 14) A 4-year-old boy with a karyotype of 47,XY+21 develops pancytopenia, lethargy, numerous bruises, pallor, and fever. Subsequent examination of the bone marrow is most likely to show which of the following? Aplasia Excess lymphoblasts Megaloblastosis Parvovirus inclusions Ringed sideroblasts 15) A 40-year-old woman with breast cancer is brought to the physician because of a 1-week history of progressive loss of peripheral vision to the left in both eyes. On examination of each eye, she is unable to count fingers or detect finger movements on the left. An MRI of the brain is most likely to show a metastatic tumor in which of the following locations? Occipital lobe Optic chiasm Optic nerve Parietal lobe Temporal lobe 16) A 13-year-old girl is brought to the physician by her mother because of a 5-month history of behavioral problems. The mother states that her daughter alternates from being sad and socially withdrawn to being extremely angry and aggressive. After further evaluation, a diagnosis of bipolar disorder is made. Treatment with valproic acid, which inhibits histone deacetylase, is started. This drug is most likely to affect which of the following processes in this patient? mRNA splicing Polyadenylation Post-translational processing Transcription Translation 17) A 62-year-old man undergoes cystoscopy for investigation and removal of a suspected renal calculus. During the procedure, a cystoscope is inserted through the penis and into the bladder. Slit- like openings are then visualized near the lateral ends of the top of the trigone region of the bladder. Which of the following best describes these structures? Ducts of the bulbourethral glands Ejaculatory ducts Internal urethral orifices Ureteric orifices Uvulae of the bladder 18) A 2-month-old male infant with a ventricular septal defect is brought to the physician because of poor weight gain and a 5-day history of rapid breathing and difficulty feeding. The diagnosis of congestive heart failure is made, and an operation to place a synthetic patch across the septal defect is planned. Compared with heart pressures prior to the operation, which of the following heart pressures are most likely 1 month after a successful operation? Left Ventricular Pressure Right Ventricular Pressure Left Atrial Pressure Increased increased no change Increased decreased decreased No change increased decreased Decreased no change decreased Decreased decreased increased 19) A 45-year-old man comes to the physician for a follow-up examination 2 weeks after beginning treatment with hydrochlorothiazide for hypertension. He says that he feels faint, light-headed, and dizzy when he gets out of bed and when he rises from a seated position too quickly. His pulse is 75/min, respirations are 12/min, and blood pressure is 130/85 mm Hg while supine. Physical examination shows no abnormalities. Which of the following sets of changes best characterizes changes in the cardiovascular system as this man goes from the supine to the standing position? Carotid Sinus Cerebral Venous Return Baroreceptor Activity Blood Flow ↑ ↑ ↑ ↑ ↑ ↓ ↑ ↓ ↑ ↑ ↓ ↓ ↓ ↑ ↑ ↓ ↑ ↓ ↓ ↓ ↑ ↓ ↓ ↓ 20) A 62-year-old man comes to the physician because of a productive cough, night sweats, and temperatures to 37.8°C (100°F) during the past 2 weeks. A PPD skin test is positive, but culture of sputum shows only normal flora after 2 days. A biopsy of an enlarged lymph node is done. The causal organisms are most likely to be present in which of the following cells in the specimen? Epithelial cells Fibroblast Macrophages Neutrophils T lymphocytes 22) A newborn delivered at 28 weeks' gestation is in severe respiratory distress. The immature alveoli of this newborn's lungs have a diminished ability to serve as sites of effective gas exchange. An increase in which of the following parameters best explains this finding? Alveolar radii Lung compliance Lung elastic recoil Pleural pressure Surfactant secretion 23) A study is conducted to evaluate the effectiveness of cough medication in children. A total of 120 patients under the age of 12 years who have been diagnosed with a viral upper respiratory tract infection are randomly selected to receive dextromethorphan or no treatment. The participants were recruited from several emergency departments in an urban area. Parents complete a rating scale for their children's cough at baseline and 1 day after receiving dextromethorphan or nothing. The results show a statistically significant improvement in cough scores for patients receiving dextromethorphan (p=0.04) compared with children receiving no treatment. Which of the following is the most significant limitation to the internal validity of this study? Lack of blinding between groups Limited number of participants Marginal statistical significance Study location limited to only emergency departments Subjective nature of the survey instrument 24) A 75-year-old woman is brought to the physician because of a 1-day history of fever and back pain. Her temperature is 39.5°C (103.1°F), pulse is 111/min, respirations are 32/min, and blood pressure is 115/79 mm Hg. Physical examination shows left-sided costovertebral angle tenderness. Laboratory studies show a leukocyte count of 17,000/mm3 (with 9% bands) and pyuria. Urine cultures grow Escherichia coli. It is determined that the patient's fever is partially due to interleukin-6 (IL-6), which was induced by the IL-1 produced during the immune response. Which of the following best describes the role of IκB in the nuclear factor-kappa B (NF-κB) signal transduction pathway from IL-1 binding to IL-6 induction in this patient? Attaches to cytokine receptor Facilitates proteolytic cleavage of NF-κB dimers Phosphorylates NF-κB Releases NF-κB after undergoing phosphorylation Translocates to the nucleus after undergoing phosphorylation 25) A 22-year-old basketball player who uses cocaine recreationally suddenly develops severe shortness of breath after a routine scrimmage. Physical examination shows respirations of 25/min. The left lung field is hyperresonant and breath sounds are diminished. Which of the following is the most likely diagnosis? Acute respiratory distress syndrome Cor pulmonale Flash pulmonary edema Pleural effusion Pneumothorax 26) Which of the following best explains why deoxygenated blood can carry more carbon dioxide for a given Pco2 than oxygenated blood? Deoxyhemoglobin does not bind to 2,3-bisphosphoglycerate as efficiently as oxyhemoglobin Deoxyhemoglobin has a lower capacity to form carbamino compounds than oxyhemoglobin Deoxyhemoglobin has a lower pKa than oxyhemoglobin Deoxyhemoglobin is a better buffer of hydrogen ions than oxyhemoglobin Oxygen and carbon dioxide compete for the same binding site in hemoglobin Oxyhemoglobin binds nitric oxide with a higher affinity than deoxyhemoglobin 27) An investigator wishes to test the hypothesis that a number of CD44 splice variants correlate with aggressive disease progression and risk of relapse in patients with Hodgkin disease, especially if the variant protein CD44v10 is expressed. The investigator plans to retrospectively screen archives of fresh frozen tissue and paraffin-embedded biopsy specimens from documented Hodgkin disease patients. Assuming the necessary reagents are available, which of the following methods would be the most effective in screening the archives for the variant protein? Affinity column chromatography Density gradient centrifugation Immunohistochemistry Northern blot Polymerase chain reaction (PCR) Reverse transcriptase PCR Southern blot 28) A 3-year-old boy with chronic otitis media is scheduled to undergo bilateral myringotomies with placement of polyethylene tubes under general anesthesia. He is asleep after only three to five breaths of inhalational anesthetic delivered by face mask. Which of the following properties of the anesthetic is the most likely explanation for the rapid onset of action? High blood solubility High hepatic metabolism High minimum alveolar concentration Low blood solubility Low hepatic metabolism Low minimum alveolar concentration 29) A 4-day-old male newborn who was born at home is brought to the emergency department because of respiratory distress and cyanosis. The mother reports that she found him in his crib not breathing. He began to breathe again after she picked him up. Examination shows a narrow thorax. His ears have periauricular skin tags. He also has micrognathia, glossoptosis, a mandibular cleft, and a short palate. Tracheostomy relieves his respiratory distress. Defects such as these are consistent with altered development of which of the following pharyngeal arches? First Second Third Fourth Sixth 30) An investigator is designing a study to compare a new behavioral program for attention- deficit/hyperactivity disorder (ADHD) with the standard behavioral modification program for this disorder. Because ADHD is more common among boys, girls and boys are randomized into the two treatment groups separately. Which of the following types of treatment allocation is most likely being used in this study? Alternation Open Outcome-adapted Simple random assignment Stratification 31) A 22-year-old man is brought to the emergency department 30 minutes after he was hit by an automobile while crossing the street. The bumper of the automobile struck him on the lateral side of his left leg. On physical examination, he is unable to dorsiflex the left foot. Inversion and eversion of the foot are normal. Which of the following nerves is most likely damaged in this patient? Deep fibular (peroneal) Femoral Lateral femoral cutaneous Obturator Tibial 32) A 43-year-old man comes to the physician because of progressive shortness of breath during the past year. He is a stockbroker and has had no environmental exposure to toxins. When asked about his smoking habits, he says he smoked marijuana in college and afterwards, until the age of 28 years. He has not smoked any substance since then. He was adopted as a child and does not know his family history. His respirations are 29/min. Diffuse wheezing is heard on auscultation of the chest. Abdominal examination shows mild hepatomegaly. Pulmonary function tests show a decreased FEV1 and decreased diffusing capacity. A chest x-ray shows hyperinflation consistent with air trapping mostly in the lung bases. A CT scan of the chest is shown. Which of the following is the most likely cause of these findings? Asthma Continuous marijuana use Diffuse idiopathic interstitial fibrosis History of smoking marijuana sprayed with a toxic herbicide Inherited protease deficiency 33) Vision screening is performed on 4000 people who are older than 65 years. Cataracts are found in 200. Which of the following best describes the annual incidence of cataracts in this population? 0.2% 0.5% 2% 5% Unable to calculate due to insufficient data 34) A healthy 25-year-old man participates in a study of muscle function. The electrophysiologic observations made on a muscle biopsy specimen are shown. Via iontophoresis, 1 μM acetylcholine (ACh) was applied to the muscle surface. Extracellular Ca2+ concentration was decreased to prevent end-plate potentials from acting as a suprathreshold for muscle action potentials. epp Amplitude mepp Amplitude Response (in mV) to ACh (in mV) (in mV) (1 μM) Normal muscle 10 1 1 epp=end-plate potential; mepp= miniature epp Based on these findings, which of the following electrophysiologic characteristics is expected in a muscle biopsy specimen from a patient with acute botulism? epp Amplitude mepp Amplitude Response (in mV) to ACh (in mV) (in mV) (1 μM) A)1 0.5 0.5 B)1 1 1 C)2 0.2 0.2 D)5 0.2 0.1 E)15 1 1 35) A 6-year-old boy with severe mental retardation and choreoathetosis is being evaluated for Lesch- Nyhan syndrome. Leukocytes are obtained for an assay of hypoxanthine-guanine phosphoribosyltransferase at saturating concentrations of phosphoribosylpyrophosphate and varying concentrations of guanine. Results of the data are shown. Guanine Velocity (Patient) (mM) μmoles/min 0.06 0.26 0.2 0.60 0.5 1.1 1.0 1.4 2.0 1.7 4.0 1.8 8.0 1.9 20 2.0 200 2.0 Which of the following is the approximate Michaelis constant [ Km] (in mM) for guanine in this patient? < 0.06 0.2 to 0.5 0.5 to 1.0 1.0 to 2.0 8.0 to 20 36) A previously healthy 26-year-old woman comes to the physician because of a 1-day history of severe rectal pain with no bleeding. She takes no medications and does not smoke cigarettes or drink alcohol. She has been sexually active, and she and her partner use condoms inconsistently. A photograph of the rectal area is shown. Which of the following is the most likely diagnosis? Anal carcinoma Anal fissure External hemorrhoid Human papillomavirus infection 37) A 64-year-old man comes to the physician because of a 2-month history of progressive shortness of breath with minimal activity. His temperature is 37°C (98.6°F), respirations are 30/min, and blood pressure is 125/80 mm Hg. Physical examination shows clubbing of the fingers. His total lung capacity is decreased. A chest x-ray shows a coarse reticular pattern. Which of the following is the most likely underlying cause of these findings? Constriction of the terminal bronchioles Destruction of the alveolar walls Increased fibrosis in the interstitium Increased mucus secretion in the bronchioles Loss of elastic support to the walls of the bronchioles 38) A 55-year-old man comes to the physician because of a 2-month history of increasing difficulty swallowing and regurgitation of undigested food. He also has noticed unusual rumbling sounds in his voice that he feels originate in his neck. Physical examination shows halitosis. A videofluoroscopic swallowing study shows a 4-cm, posterior midline pouch protruding between the thyropharyngeus and cricopharyngeus portions of the inferior pharyngeal constrictor muscle. These muscles are most likely innervated by which of the following nerves? Glossopharyngeal nerve Hypoglossal nerve Motor fibers from the vagus nerve Parasympathetic fibers from the vagus nerve Sympathetic fibers from the superior cervical ganglion 39) A 20-year-old competitive cyclist taking recombinant human erythropoietin has polycythemia. Which of the following is the most likely cause? Early release of reticulocytes from bone marrow Increased erythrocyte life span Proliferation of bone marrow stem cells Proliferation of erythroid precursors 40) A 6-year-old girl is brought to the physician for a follow-up examination. She has been receiving treatment with potassium citrate monohydrate since the diagnosis of renal tubular acidosis was made at the age of 2 years. Physical examination shows no abnormalities. Serum studies show: Na+ 142 mEq/L K+ 3.5 mEq/L Cl– 115 mEq/L HCO3– 18 mEq/L Urea nitrogen 9 mg/dL Creatinine 0.9 mg/dL A defect in renal ammoniagenesis is suspected. Which of the following substrates is the most likely source of ammonia production in this patient? Creatinine Glutamine Glycine Leucine Urea 41) A 35-year-old woman, gravida 1, para 1, comes to the physician because of a 2-month history of generalized weakness and fatigability that are significantly exacerbated by exercise. The symptoms began during her pregnancy. Physical examination shows diplopia. There is reduction in arm muscle strength with repetitive movement, but preservation of deep tendon reflexes. A therapeutic trial of oral pyridostigmine provides symptomatic relief. Serum studies will most likely show antibodies to which of the following proteins? Acetylcholinesterase Muscarinic acetylcholine receptor Myeloperoxidase Nicotinic acetylcholine receptor 42) A 2-year-old girl is brought to the emergency department 20 minutes after she accidently ingested insecticide. Her father reports that he was spraying his lawn and accidentally left the open bottle on the ground. His daughter walked over, picked up the bottle, and started to drink, but then she immediately threw down the bottle. She is in moderate respiratory distress. Her respirations are 18/min and shallow. Pulse oximetry on room air shows an oxygen saturation of 82%. Physical examination shows copious oral secretions. In addition to securing an airway, the most appropriate immediate step is administration of which of the following medications? Atropine Diphenhydramine Physostigmine Pralidoxime Succinylcholine 43) Following a stroke, a 68-year-old man has a language problem. His speech is fluent but contains many grammatical errors, word substitutions, and neologisms. He is unable to repeat words after the examiner and is apparently unable to comprehend other verbal requests. Which of the following labeled sites on the photograph of the left hemisphere is most likely to be damaged? GROSS ANATOMY BRAIN IMAGE 44) A 30-year-old man is brought to the physician by his wife because of a 1-day history of irrational behavior and severe abdominal pain. He began treatment with trimethoprim-sulfamethoxazole for a urinary tract infection 3 days ago. His urine turned a light burgundy color during a similar episode 1 year ago. His mother and his maternal grandfather have had similar symptoms. He appears anxious and restless. His pulse is 96/min. Physical examination shows diaphoresis. Serum studies show increased concentrations of 5-aminolevulinic acid (δ-ALA) and porphobilinogen. The physician suspects that a mutant allele is causing decreased activity of an enzyme involved in heme biosynthesis. This enzyme is most likely which of the following? δ-ALA dehydratase δ-ALA synthase Ferrochelatase Porphobilinogen deaminase Uroporphyrinogen decarboxylase 45) A 25-year-old woman with a history of rheumatic fever and mitral valve dysfunction comes to the physician because of a 2-week history of fever and fatigue. She underwent a root canal procedure 1 month ago, before which she had taken a single dose of amoxicillin. Her temperature is 38.4°C (101.2°F). A grade 4/6 blowing murmur is heard on auscultation under the left axilla. A photomicrograph of a Gram stain of the organism recovered from a blood culture specimen is shown. On blood agar plates, the organism shows alpha hemolysis. Which of the following is the most likely causal organism? Enterococcus faecalis Group A beta-hemolytic streptococci Staphylococcus aureus Streptococcus mitis Streptococcus pneumoniae 46) A 56-year-old woman comes to the physician because of a 3-week history of progressive difficulty swallowing. When her symptoms began, she felt that bread and larger pieces of meat were getting stuck in her chest before passing through completely, but now she is having difficulty swallowing all foods and some liquids. Physical examination shows normal oral coordination and a nontender abdomen. An x-ray obtained after a barium swallow shows an irregular mass at the gastroesophageal junction. A photomicrograph of a biopsy specimen obtained via esophagogastroduodenoscopy is shown. Chronic infection with which of the following pathogens is the most likely cause of the histologic findings in this patient? Clonorchis sinensis Cytomegalovirus Entamoeba histolytica Helicobacter pylori Mycobacterium tuberculosis 47) A 45-year-old man comes to the physician for an initial examination. He has not been examined by any physician during the past 30 years because he has not felt ill. He says that he feels well at this visit. He appears relaxed. Cardiac examination shows a grade 2/6, holosystolic murmur that is best heard at the left axillary line. Which of the following is the most likely cause of this finding? Aortic regurgitation Aortic stenosis Mitral regurgitation Mitral stenosis Tricuspid regurgitation Tricuspid stenosis 48) A 71-year-old woman comes to the physician for a follow-up examination. She has a 30-year history of type 2 diabetes mellitus currently treated with insulin. She is 160 cm (5 ft 3 in) tall and weighs 59 kg (130 lb); BMI is 23 kg/m2. Her blood pressure is 116/78 mm Hg. This patient is most likely to have an endogenous fasting serum insulin concentration (N=5–20 μU/mL) closest to which of the following? 1 μU/mL 10 μU/mL 20 μU/mL 40 μU/mL 80 μU/mL 49) A 22-year-old woman is found to be HIV positive after sexual contact with a partner with HIV infection. Combination therapy with lamivudine (3TC), ritonavir/lopinavir, and zidovudine (AZT) is initiated. Three months later, genomic typing shows that her HIV strain has become resistant to ritonavir/lopinavir. The cause of this resistance is most likely the acquisition of a mutation in a gene that is critical for which of the following viral processes? Adsorption and penetration Early protein synthesis Genome integration Late protein synthesis Nucleic acid synthesis Packaging and assembly Protein processing Release Uncoating 50) A 43-year-old man with a 10-year history of alcoholism comes to the physician because of a change in skin color. The patient says that he has been taking two extra-strength acetaminophen tablets every 4 to 6 hours for the past 3 days for a severe headache. Physical examination shows jaundice. Laboratory studies show an increased prothrombin time and a markedly increased serum AST activity. Alteration in which of the following metabolites within hepatocytes is most likely associated with his illness? Decreased glucuronide conjugates Decreased glutathione Decreased NAD+ Decreased NADH Increased glucuronide conjugates Increased glutathione Increased NAD+ Increased NADH 51) A 25-year-old primigravid woman at 12 weeks' gestation comes to the physician 1 hour after she passed a small amount of tissue vaginally at home. Gross examination of this tissue shows an obvious fetus. Microscopic examination of tissue obtained via dilatation and curettage shows chorionic villi with focal edema and trophoblastic proliferation. Which of the following is the most likely diagnosis? Choriocarcinoma Complete hydatidiform mole Partial hydatidiform mole Placenta accreta Placental site trophoblastic tumor 52) A 67-year-old man is brought to the physician because of a 2-day history of double vision and drooping of his left eyelid. He has no history of trauma to the eye. His vital signs are within normal limits. Physical examination shows ptosis of the left eyelid. When the eyelid is raised manually, the eye is fixed in the out position and the pupil is dilated. The visual acuity of the left eye is within normal limits. Which of the following is the most likely cause of these findings? Aneurysm of the posterior communicating artery Compression of the superior cervical ganglion Damage to the trochlear nerve Occlusion of the scleral venous sinus Tumor of the optic nerve 53) A 2-month-old male infant is brought to the emergency department by his mother because of a 2- day history of generalized tonic-clonic seizures, myoclonus, and hiccuping. His mother says he has fed poorly and has been floppy since birth. Physical examination shows decreased deep tendon reflexes and hypotonia. A signaling defect in an inhibitory neurotransmitter is suspected. Which of the following neurotransmitter receptors most likely contains the defect? Acetylcholine receptor Aspartic acid receptor Epinephrine receptor Glutamate receptor Glycine receptor 54) A 47-year-old man is admitted to the hospital for treatment of a myocardial infarction. On admission, pulse oximetry on 30% oxygen shows an oxygen saturation greater than or equal to 95%. Three hours later, the patient develops shortness of breath. Pulse oximetry now shows an oxygen saturation of 90%. Crackles are heard at the lung bases, and a grade 2/6 systolic murmur is heard. Arterial blood gas analysis on 30% oxygen shows: pH 7.41 Pco2 36 mm Hg Po2 60 mm Hg Which of the following is the most likely cause of this patient's hypoxemia? Decreased alveolar ventilation Decreased erythrocyte transit time in pulmonary capillaries Decreased lymphatic drainage Increased permeability of pulmonary capillaries Increased plasma colloid osmotic pressure Increased pulmonary capillary pressure 1. A 3-year-old girl is brought to the physician because of a 1-year history of short stature. She has no history of major medical illness. She is below the 3rd percentile for height and at the 10th percentile for weight. Physical examination shows coarse facial features and contractures of the large joints. X-rays show dysostosis multiplex. Plasma lysosomal enzyme analysis shows increased B- hexosaminidase, [3-glucuronidase, B-galactosidase, and d-fucosidase activities. Which of the following mechanisms is the most likely cause of the lysosomal enzyme findings in this patient‘? A) Abnormal targeting of these enzymes to lysosomes B) Deficiency of the other lysosomal enzymes in the cytoplasm C) Degradation of these enzymes within the cytoplasm D) Degradation of these enzymes within the lysosomes E) Storage of these enzymes within the cytoplasm 2. A 20-year-old competitive cyclist taking recombinant human erythropoietin has polycythemia. Which of the following is the most likely cause? A) Early release of reticulocytes from bone marrow B) Increased erythrocyte life span C) Proliferation of bone marrow stem cells D) Proliferation of erythroid precursors 3. A 25-year-old primigravid woman at 12 weeks‘ gestation comes to the physician 1 hour after she passed a small amount of tissue vaginally at home. Gross examination of this tissue shows an obvious fetus. Microscopic examination of tissue obtained via dilatation and curettage shows chorionic villi with focal edema and trophoblastic proliferation. Which of the following is the most likely diagnosis? A) Choriocarcinoma B) Complete hydatidiform mole C) Partial hydatidiform mole D) Placenta accreta E) Placental site trophoblastic tumor 4. A study is conducted to evaluate the effectiveness of cough medication in children. A total of 120 patients under the age of 12 years who have been diagnosed with a viral upper respiratory tract infection are randomly selected to receive dextromethorphan or no treatment. The participants were recruited from several emergency departments in an urban area. Parents complete a rating scale for their children's cough at baseline and 1 day after receiving dextromethorphan or nothing. The results show a statistically significant improvement in cough scores for patients receiving dextromethorphan (p=0.04) compared with children receiving no treatment. Which of the following is the most significant limitation to the internal validity of this study? A) Lack of blinding between groups B) Limited number of participants C) Marginal statistical significance D) Study location limited to only emergency departments E) Subjective nature of the survey instrument 5. A 40-year-old woman with breast cancer is brought to the physician because of a 1-week history of progressive loss of peripheral vision to the left in both eyes. On examination of each eye, she is unable to count fingers or detect finger movements on the left. An MRI of the brain is most likely to show a metastatic tumor in which of the following locations? A) Occipital lobe B) Optic chiasrn C) Optic nerve D) Parietal lobe E) Temporal lobe 6. A 54-year-old woman comes to the physician because of a 2-week history of heavy vaginal bleeding. Menopause occurred 5 years ago. She underwent a mastectomy 3 years ago for estrogen receptor-positive breast carcinoma with axillary lymph node involvement. She has been taking tamoxifen, a drug that blocks estrogen receptors in breast tissue, for the past 3 years. Biopsy of endometrial tissue shows hyperplasia. Which of the following best explains the development of endometrial hyperplasia in this patient? A) Endometrial tissues become hyperplastic in the absence of estrogen B) The patient has been noncompliant in taking the estrogen receptor antagonist C) The patient's breast cancer has metastasized to the patient's uterus D) The patient's breast cancer is associated with an estrogen-secreting ovarian cancer E) The patient's medication has an agonist effect on endometrial estrogen receptors F) The patient's serum estrogen concentrations are increased due to ectopic production of human chorionic gonadotropin 7. A 2-month-old girl is brought to the physician for a follow-up examination 2 days after a complete blood count was found to be abnormal. Her blood group is A, Rh-positive. She was born at term to a healthy 23-year-old woman whose blood group is O, Rh-positive. Birth weight was 3890 g (8 lb 9 02). Pregnancy and delivery were uncomplicated. The patient has been active and feeding well. Her temperature is 37.6°C (99.7°F), pulse is 120/min, and respirations are 30/min. Physical examination shows no abnormalities. Laboratory studies done since birth are shown. Patient Age 24 Hours Hemoglobin (g/dL) (N=1 1-15) 16.9 Hematocrit (%) 51 Leukocyte count (Imm3) (N=5000-19,500) 17,200 Segmented neutrophils 4% Eosinophils 4% Basophils 4% Lymphocytes 69% Monocytes 19% Platelet count (Imm 3) (N=150,000—400,000) 185,000 1 Week Hemoglobin (g/dL) (N=1 1-15) 15.7 Hematocrit (%) 49 Leukocyte count (Imm3) (N=5000-19,500) 15,600 Segmented neutrophils 3% Eosinophils 4% Basophils 2% Lymphocytes 73% Monocytes 18% Platelet count (Imm 3) (N=150,000—400,000) 198,000 1 Month Hemoglobin (g/dL) (N=1 1-15) 12.7 Hematocrit (%) 39 Leukocyte count (Imm3) (N=5000-19,500) 12,900 Segmented neutrophils 5% Eosinophils 2% Basophils 1% Lymphocytes 77% Monocytes 15% Platelet count (Imm 3) (N=150,000—400,000) 205,000 Which of the following is the most likely diagnosis? A) Alloimmune hemolytic disease of the newborn B) Congenital cytomegalovirus infection C) Congenital neutropenia D) DiGeorge syndrome E) Severe combined immunodeficiency 8. A 13-year-old boy is scheduled to receive chemotherapy for a leukemia that has the histologic features of malignant lymphocytes. This neoplasm is further typed for cell surface and intracellular markers specific for lymphocyte subsets. The neoplastic cells do not express the following markers: CD4, CDB, surface lgM, surface lgG, cytoplasmic lgM and u-heavy chain, cytoplasmic I96, and Y-heavy chain. The leukemic cells express class I MHC molecules and show rearrangement of the T-lymphocyte receptor B-chain gene D and J segments. Which of the following is the normal counterpart of these malignant lymphocytes? A) Activated cytolytic effector T lymphocytes in the circulation 8) Mature lgM-secreting B lymphocytes in the lymph node C) Mature immunoglobulin-secreting plasma cells in the lymph node D) Pre-B lymphocyte progenitor of mature B lymphocytes in the bone marrow E) T-Iymphocyte thymocytes localized to the thymic cortex 9. A 56-year-old woman is brought to the emergency department by her husband because of fever and shortness of breath for 2 hours. Her husband says that she also has had urinary frequency and pain with urination for 2 days. She is in respiratory distress. Her temperature is 38.7°C (101 pulse is 120/min, respirations are 30/min, and blood pressure is 80/50 mm Hg. Which of the following components of the causal organism is the most likely cause of the hypotension? A) Flagellar H antigen B) Lipopolysaccharide C) P pilus D) Polysaccharide capsule E) Shiga-Iike cytotoxin 10. A newborn undergoes surgical repair of esophageal atresia. Pathologic examination of resected tissue shows that the esophagus is and has no lumen. Which of the following embryonic germ layers is the most likely origin of the cells that fill the lumen? A) Endoderm B) Intermediate mesoderm C) Neural crest D) Paraxial mesoderm E) Surface ectoderm 11. A 50-year-old man comes to the physician because of a 2-week history of progressive shortness of breath while climbing stairs to his office every morning. He reports no other problems, but he is concerned because his father had a major myocardial infarction at the age of 52 years. His pulse is 110/min and regular, respirations are 16/min, and blood pressure is 135/95 mm Hg. The lungs are clear to auscultation. Cardiac examination shows normal heart sounds with a physiologic split of 82. Stress echocardiography shows hypokinesis of the posterior left ventricle with increasing activity levels. Which of the following is the most likely cause of the posterior left ventricular findings in this patient? A) Disruption of the sympathetic nerves to the left ventricle B) Extravascular compression of the coronary arteries C) Increased left ventricular end-diastolic pressure D) Increased myocardial oxygen consumption E) Stenosis of the right coronary artery 12. A 75-year-old woman comes to the physician because of a 3-month history of an enlarging lesion on her face. Physical examination shows a 1.5-cm, brown-black, mottled, scaly lesion with irregular borders. Microscopic examination of a biopsy specimen of the lesion shows atypical melanocytes spread along the basilar layer of the epidermis. Which of the following is the most likely cause of these findings? A) Acanthosis nigricans B) Actinic keratosis C) Compound nevus D) Lentigo maligna E) Seborrheic keratosis 13. A 59-year-old man comes to the physician because of a 10-day history of shortness of breath, fatigue, and cough. His respirations are 12/min. The lungs are clear to auscultation and percussion. Laboratory studies show: Hemoglobin 11.8 g/dL Hematocrit 36% Leukocyte count 146,400/mm3 Segmented neutrophils 47% Bands 12% Eosinophils 1% Basophils 5% Lymphocytes 2% Monocytes 2% Metamyelocytes 7% Myelocytes 18% Prornyelocytes 2% Other cells 4% Platelet count 804,000/mm3 Which of the following is the most appropriate pharmacotherapy for this patient? A) Anti-interleukin-S (IL-5) B) Cytarabine C) Hydroxyurea D) lmatinib E) Interferon alfa 14. An investigator wishes to test the hypothesis that a number of CD44 splice variants correlate with aggressive disease progression and risk of relapse in patients with Hodgkin disease, especially if the variant protein CD44v10 is expressed. The investigator plans to retrospectively screen archives of fresh frozen tissue and paraffin-embedded biopsy specimens from documented Hodgkin disease patients. Assuming the necessary reagents are available, which of the following methods would be the most effective in screening the archives for the variant protein? A) Affinity column chromatography B) Density gradient oentrifugation C) Immunohistochemistry D) Northern blot E) Polymerase chain reaction (PCR) F) Reverse transcriptase PCR G) Southern blot 15. A 75-year-old woman is brought to the physician because of a 1-day history of fever and back pain. Her temperature is 395°C (103.1°F), pulse is 111/min, respirations are 32/min, and blood pressure is 115/79 mm Hg. Physical examination shows left-sided costovertebral angle tenderness. Laboratory studies show a leukocyte count of 17,000/mm3 (with 9% bands) and pyuria. Urine cultures grow Escherichia coli. It is determined that the patient's fever is partially due to interleukin-6 (IL-6), which was induced by the lL-1 produced during the immune response. Which of the following best describes the role of IKB in the nuclear factor-kappa B (NF-KB) signal transduction pathway from IL-1 binding to IL-6 induction in this patient? A) Attaches to cytokine receptor B) Facilitates proteolytic cleavage of NF-KB dimers C) Phosphorylates NF-KB D) Releases NF-KB after undergoing phosphorylation E) Translocates to the nucleus after undergoing phosphorylation 16. Which of the following best explains why deoxygenated blood can carry more carbon dioxide for a given Poo2 than oxygenated blood? A) Deoxyhemoglobin does not bind to 2,3-bisphosphoglycerate as efficiently as oxyhemoglobin B) Deoxyhemoglobin has a lower capacity to form carbamino compounds than oxyhemoglobin C) Deoxyhemoglobin has a lower pKa than oxyhemoglobin D) Deoxyhemoglobin is a better buffer of hydrogen ions than oxyhemoglobin E) Oxygen and carbon dioxide compete for the same binding site in hemoglobin F) Oxyhemoglobin binds nitric oxide with a higher affinity than deoxyhemoglobin 17. A 3-year-old boy with chronic otitis media is scheduled to undergo bilateral myringotomies with placement of polyethylene tubes under general anesthesia. He is asleep after only three to five breaths of inhalational anesthetic delivered by face mask. Which of the following properties of the anesthetic is the most likely explanation for the rapid onset of action? A) High blood solubility B) High hepatic metabolism C) High minimum alveolar concentration D) Low blood solubility E) Low hepatic metabolism F) Low minimum alveolar concentration 18. An investigator is studying Helicobacter pylori strains isolated from the same patient over several years. The immune response to proteins produced by these strains is observed. It is found that the original H. pylori isolate from the patient expresses one protein recognized by the patient's antibodies, but subsequent isolates do not express this protein. Sequencing of the gene encoding the protein from the original and subsequent isolates is done. It is found that subsequent isolates have nine consecutive cytidine residues, whereas the original isolate has eight. The results are shown. Original isolate: ...ACC CCC CCC ACT CAA ATT GAA CCT AGC... ...Thr Pro Pro Thr Gln lle Glu Pro Ser... Subsequent isolates: ACC CCC CCC CAC TCA AAT TGA ACC TAG ...Thr Pro Pro His Ser Asn STOP Which of the following mechanisms best explains this genetic change? A) Crossing over B) DNA excision repair C) Slipped-strand mispairing D) Thymidine dimer formation E) Transposon insertion 19. During a series of normal skeletal muscle twitches, the ATP concentration does not fall appreciably because of which of the following? A) ATP is hydrolyzed only during relaxation B) ATP is quickly regenerated from creatine phosphate C) Creatine phosphate is hydrolyzed by myosin D) The initial ATP concentration is high 20. A 6-year-old boy with severe mental retardation and choreoathetosis is being evaluated for Lesch- Nyhan syndrome. Leukocytes are obtained for an assay of hypoxanthine-guanine phosphoribosyltransferase at saturating concentrations of phosphoribosylpyrophosphate and varying concentrations of guanine. Results of the data are shown. Guanine Velocity (Patient) (mM) pmoleslmin 0.06 0.26 0.2 0.60 0.5 1.1 1.0 1.4 2.0 1.7 4.0 1.8 8.0 1.9 20 2.0 200 2. A 51 year old woman undergoes dilatation and curettage for DUB.During the induction of anasthesia,Isoflurane is administered along with a second compound.30 minutes later,she develops a temperature of 40.5 degree celsius and muscle rigidity.Simultaneous administration of which of the following compounds is the most likely cause of the adverse effect in this patient? A.Bupivacaine B.ketamine C.midazolam D.Neostigmine E.Succinyl choline 1) A newborn delivered at 28 weeks' gestation is in severe respiratory distress. The immature alveoli of this newborn's lungs have a diminished ability to serve as sites of effective gas exchange. An increase in which of the following parameters best explains this finding? A) Alveolar radii B) Lung compliance C) Lung elastic recoil D) Pleural pressure E) Surfactant secretion ----------------- 2) A 38-year-old man comes to the physician because of progressive heartburn over past 6 months. He states that liquid antacids, at least twice daily, no longer relieve the pain. He has modified his diet, decreased his coffee and citrus intake, and has raised the head of his bed, Physical examination shows no acute abnormalities. Upper endoscopy shows marked erythema of the distal 6 cm of esophagus. three linear erosions are noted. Biopsy specimens show marked inflammation of squamous epithelium. Which of the following is the most appropriated pharmacotherapy at this time? A) Calcium carbonate B) Cimetidine C) Metoclopramide D) Omeprazole E) Sucralfate -------------------- 3) A 52-year-old man is brought to the physician 3 days after the sudden onset of blindness of the left eye. He does not have any eye pain. Funduscopy of the left eye shows a pale, opaque fundus and a bright red fovea centralis. Visual field testing shows a dense scotoma of the entire visual field of the left eye; testing of the right eye shows no abnormalities. At a follow-up examination 6 months later, the patient remains blind in the left eye. If the left eye is illuminated, which of the following reactions is mostly likely in the right pupil of this patient? A) Constriction because the left optic tract is binocular B) Constriction because projections to the Edinger-Westphal nucleus are bilateral C) Dilation because the posterior commissure is intact D) Dilation because the right superior cervical ganglion is intact E) No constriction because the left ciliary nerve has been permanently damaged F) No constriction because the retinal ganglion cells in the left eye have been destroyed. ---------------- 4) A 19-year-old woman is brought to the physician by her parents because of a 1-year history of bizarre behavior, poor personal hygiene, and extended periods spent alone in her room. On mental status examination, she is thin and unkempt, with soft speech and flat affect. When asked to describe her mood, she says, "i feel fine," she adds, "But I often hear voices that on one else does." Which of the following is the most appropriate initial pharmacotherapy? A) Fluoxetine B) Lithium carbonate C) Lorazepam D) Olanzapine E) Valproic acid a previously healthy 35 yr old woman develops hypoxemia 35 minutes after ingesting a near-lethal dose of barbiturates. she has not aspirated. which of the foll sets of ABG values ( in mm HG) is most likely in this patient? --------------pO2--------------pCO2---------(A-a)O2 a)--------------40---------------50--------------35 b)--------------40--------------60---------------40 c)--------------50--------------25----------------10 d)--------------50--------------80--------------10 e)--------------60--------------35----------------25 Q1)A 39-year-old woman comes to the physician for an initial evaluation after moving to a new city. She has a 25-year history of multiple symptoms, including band-like headaches; blurred vision; pain in her neck, back, arms, and legs; nausea; irregular bowel movements; urinary urgency; difficulty attaining an orgasm; and excessive menstrual bleeding. She says that she has seen multiple physicians since she was a teenager, undergone many diagnostic tests, and has been admitted to the hospital numerous times, but no abnormalities have ever been found. She brings a copy of her medical records with her, which confirms the lack of a definitive diagnosis. Which of the following is the most appropriate statement by the physician? A) "I'd like to assess the symptom causing you the most distress and schedule monthly follow-up appointments." B)"I'd like to do a diagnostic work-up for acute intermittent porphyria." C)"I'm sorry, but I think that you are malingering and most likely seeking drugs." D)"Tell me about any emotional trauma you may have experienced during childhood." E)"Your symptoms could be caused by stress; I would like you to undergo a psychiatric examination." Q2) Influenza virus strains are characterized by differing sensitivity to growth inhibition by the weak base ammonium chloride (AC). Influenza A San Francisco (SF) produces virus yields in AC-treated cells that approximate those in untreated cells, whereas influenza A New York (NY) grows poorly in AC-treated cells. To identify viral genes that segregate with differences in the sensitivity of these strains to AC, reassortant viruses are isolated by coinfecting cells with both virus strains. The ratios of the growth of the reassortant viruses in the presence and absence of AC are shown in the table. Virus Strain AC+/AC– Ratio Genome Segment (Protein) (PA) (HA) (NP) (NA) (M1/M2) Parental NY 0.005 NY NY NY NY NY SF 0.8 SF SF SF SF SF Reassortant NY-SF/1 1.0 SF SF NY NY SF NY-SF/2 0.8 NY SF NY SF SF NY-SF/3 0.8 SF SF SF SF NY NY-SF/4 0.5 NY SF SF NY NY NY-SF/5 0.4 NY SF SF SF NY NY-SF/6 0.008 NY NY NY SF SF NY-SF/7 0.004 SF NY NY SF SF NY-SF/8 0.002 SF NY NY SF NY Which of the following viral gene segments segregates with progeny viruses that are resistant to ammonium chloride? A)HA B)M1/M2 C)NA D)NP E)PA Q3)A 3-year-old girl is brought to the physician because of a 1-year history of short stature. She has no history of major medical illness. She is below the 3rd percentile for height and at the 10th percentile for weight. Physical examination shows coarse facial features and contractures of the large joints. X-rays show dysostosis multiplex. Plasma lysosomal enzyme analysis shows increased β- hexosaminidase, β-glucuronidase, β-galactosidase, and α-fucosidase activities. Which of the following mechanisms is the most likely cause of the lysosomal enzyme findings in this patient? A)Abnormal targeting of these enzymes to lysosomes B)Deficiency of the other lysosomal enzymes in the cytoplasm C)Degradation of these enzymes within the cytoplasm D)Degradation of these enzymes within the lysosomes E)Storage of these enzymes within the cytoplasm Q3)Four days following a hike through the forest, a 40-year-old man develops multiple intensely pruritic areas on his skin similar to the lesions shown in the photograph. The involved skin was not covered by clothing during the hike. He applies a topical ointment containing 1% hydrocortisone to the affected areas and obtains moderate relief. The lesions subside over the next 1 to 2 weeks. These findings are most characteristic of which of the following conditions? image:-http://img594.imageshack.us/img594/5783/image1pg.jpg A)Arthus reaction B)Chronic granulomatous disease C)Delayed (type IV) hypersensitivity reaction D)Insect bites E)Localized anaphylaxis F)Sunburn Q4)A 3-year-old girl is brought to the physician because of a 1-month history of a lump in her upper neck. Physical examination shows a 2-cm mass in the midline. The mass moves upward with swallowing and protrusion of the tongue. Radionucleotide imaging with technetium 99m pertechnetate shows uptake in the mass. If a biopsy specimen of this mass were obtained, it would most likely show which of the following? A)Chief and oxyphil cells B)Chondroid matrix C)Serous and mucous acini D)Taste buds E)Thyroid follicles Q5)A 32-year-old man comes to the physician for his annual company-sponsored health maintenance examination. He has no hobbies. He says that he likes his work as a computer programmer because he can do much of it at home, where he lives with 15 cats. He says, "I like to be with my cats because they are my best friends. I don't have to waste any time talking to anyone." A note from his supervisor states, "We could use him on some team projects, but he refuses. Is there some medication or therapy that would help?" Which of the following is the most likely diagnosis? A)Avoidant personality disorder B)Dysthymic disorder C)Major depressive disorder D)Narcissistic personality disorder E)Obsessive-compulsive disorder F)Schizoid personality disorder G)Schizotypal personality disorder Q6)A 2-year-old girl is brought to the emergency department 20 minutes after she accidently ingested insecticide. Her father reports that he was spraying his lawn and accidentally left the open bottle on the ground. His daughter walked over, picked up the bottle, and started to drink, but then she immediately threw down the bottle. She is in moderate respiratory distress. Her respirations are 18/min and shallow. Pulse oximetry on room air shows an oxygen saturation of 82%. Physical examination shows copious oral secretions. In addition to securing an airway, the most appropriate immediate step is administration of which of the following medications? A)Atropine B)Diphenhydramine C)Physostigmine D)Pralidoxime E)Succinylcholine Q7) Following a stroke, a 68-year-old man has a language problem. His speech is fluent but contains many grammatical errors, word substitutions, and neologisms. He is unable to repeat words after the examiner and is apparently unable to comprehend other verbal requests. Which of the following labeled sites on the photograph of the left hemisphere is most likely to be damaged? Image Link :- http://img713.imageshack.us/img713/1650/image2pt.jpg 1.A 2.B 3.C 4.D 5.E 6.F 7.G 8.H 9.I 10.J Q8)A 2-month-old male infant is brought to the emergency department by his mother because of a 2- day history of generalized tonic-clonic seizures, myoclonus, and hiccuping. His mother says he has fed poorly and has been floppy since birth. Physical examination shows decreased deep tendon reflexes and hypotonia. A signaling defect in an inhibitory neurotransmitter is suspected. Which of the following neurotransmitter receptors most likely contains the defect? A)Acetylcholine receptor B)Aspartic acid receptor C)Epinephrine receptor D)Glutamate receptor E)Glycine receptor Q9)A 22-year-old woman is found to be HIV positive after sexual contact with a partner with HIV infection. Combination therapy with lamivudine (3TC), ritonavir/lopinavir, and zidovudine (AZT) is initiated. Three months later, genomic typing shows that her HIV strain has become resistant to ritonavir/lopinavir. The cause of this resistance is most likely the acquisition of a mutation in a gene that is critical for which of the following viral processes? A)Adsorption and penetration B)Early protein synthesis C)Genome integration D)Late protein synthesis E)Nucleic acid synthesis F)Packaging and assembly G)Protein processing H)Release I)Uncoating Q10) A 16-year-old girl comes to the clinic because of vaginal itching for the past 5 days. She requests that her parents not be informed about the appointment. During the examination, she says she occasionally has unprotected sexual intercourse and asks for oral contraceptives. Preliminary laboratory studies show gram-negative diplococci. Which of the following is the most appropriate next step in patient care? A)Consult a colleague for advice about treating a minor without parental consent B)Discuss with her the need for testing her for other sexually transmitted diseases C)Interview her sexual partners D)Notify her parents of the findings E)Treat her infection and refer her to social services Q11) A 72-year-old woman is admitted to the hospital because of an acute myocardial infarction. She undergoes cardiac catheterization. Angiography shows a left dominant circulation, and a 90% narrowing of the artery supplying the diaphragmatic surface and atrioventricular node of the heart. A balloon angioplasty is scheduled during which a stent will be inserted in the narrowed vessel. The catheter and the balloon must be passed through which of the following vessels (stated in order) to reach the narrowed vessel? A)Left coronary, circumflex, anterior interventricular (left anterior descending) B)Left coronary, circumflex, posterior interventricular (posterior descending) C)Left coronary, posterior interventricular (posterior descending), circumflex D)Right coronary, circumflex, anterior interventricular (left anterior descending) E)Right coronary, circumflex, posterior interventricular (posterior descending) F)Right coronary, posterior interventricular (posterior descending), circumflex Q12)An investigator is studying the age of patients when they undergo their first colonoscopy at a local physician's practice. The following are the ages of the first 10 patients analyzed: 50 years, 50 years, 50 years, 51 years, 52 years, 58 years, 65 years, 65 years, 75 years, and 84 years. Which of the following best represents the mean of this subpopulation? A)50 years B)55 years C)60 years D)65 years E)70 years Q13) A 43-year-old man with a 10-year history of alcoholism comes to the physician because of a change in skin color. The patient says that he has been taking two extra-strength acetaminophen tablets every 4 to 6 hours for the past 3 days for a severe headache. Physical examination shows jaundice. Laboratory studies show an increased prothrombin time and a markedly increased serum AST activity. Alteration in which of the following metabolites within hepatocytes is most likely associated with his illness? A)Decreased glucuronide conjugates B)Decreased glutathione C)Decreased NAD+ D)Decreased NADH E)Increased glucuronide conjugates F)Increased glutathione G)Increased NAD+ H)Increased NADH a 72 year old man with poorly controlled hypertension is brought to ER because of severe abdominal pain for 2 hrs.He appears pale and letharic. His pulse-124/min, respirations-16/min, and blood pressure- 95/60. the lungs are clear to auscultation. An S4 is present. Abdominal examination shows guarding, rigidity, and a pulsatile periumbilical mass. which of the following additional findings is most likely in this patient. a. ankle brachial indices that are within normal limits. b. aortic regurgitation c. assymmetric radial pulses d. BP readings that are higher in left upper extremity than in right e. decreased femoral pulses which of the ff supports a diagnosis of rheumatoid arthritis a) antnuclear antibodies b)autoantibdy(igm)that cross react with igg c)anti RO A 72 year old man with chronic obstructive pulmonary disease comes to the physician because of progressive shortness of breath during the past 3 months.He had smoked 1.5 packs of cigarettes daily for 52 years,but he recently decreased the amount to 1 pack daily.He is 178 cm(5 ft 10 in)tall and weighs 109 kg(240 lb);BMI is 34 kg/m2.His pulse is 80/min, respirations are 22/min, and blood pressure is 130/80 mm Hg.Pulse oximetry on room air shows an oxygen saturation of 94%.Coarse expiratory rhonchi are heard on auscultation of the chest.Breath sounds are decreased bilaterally.Which of the following interventions is most likely to slow the rate of decline in this patient's pulmonary function? A. Smoking cessation B. Tiotropium therapy C.Pulmonary rehabilitation D.Beta 2 Adrenergic agonist therapy E. Corticosteroid therapy. . During an experiment of muscle contraction,intracellular calcium is decreased after a substance is administered to a muscle preparation obtained from an experimental animal.Which of the following best explains why contraction is inhibited in this case? A. Acetylcholine release is increased B. Depolarization along T tubules is enhanced C. Myosin binding sites on actin remain covered by troponin C D.Sodium influx is increased E.Tropomyosin is detached from actin A 37 year old woman is brought to the emergency department after her husband found her unconscious. Her temperature is 36deg C(96.8degF),pulse is 128/min, and blood pressure is 70/40 mm Hg. Physical examination shows cool, pale, extremities, jugular venous distention, faint peripheral pulses, and crackles over the bottom two thirds of both lung fields. Heart sounds are normal, and there are no murmurs. She withdraws to painful stimuli in all four extremities. This patient is most likely experiencing which of the following types of shock? A. Anaphylactic B. Cardiogenic C. Hypovolemic D. Neurogenic E.Septic A 60 year old man is brought to the emergency department because of generalized weakness for 6 hours. THe onset of this symptom occurred 2 hours after he took four tablets(rather than his usual one tablet) of a medication that prevents angina pectoris. His pulse is 36/min, and blood pressure is 100/50 mm Hg. After the administration of isoproterenol, his pulse increases to 60/min.Which of the following best describes the mechanism o action of this patient's ususal chronic medication? A. Irreversible,dose dependent B. Irreversible,dose independent C. Reversible, dose dependent D. Reversible, dose independen a newbron undergoes surgical repair of esophageal atresia.pathologic examination of resected tissue shows that the esophagus is fibrotic and has no lumen. which of the foll embryonic germ layers is the most likely origin of the cells that fill the lumen? a--- endoderm b---intermediate mesoderm c-- neural crest d--- paraxial mesoderm e-- surface ectoderm 2) a 20 yr old woman comes to the physician because her menstrual period is 1 week late. menses had occured at regular 28 day intervals. she had unprotected sexual intercourse with her boyfriend 20 days ago. a urine pregnancy test is positive. which of the following best describes the stage of developmant of the embryo at this time? a) the cytotrophoblast is present, but the syncitiotrophoblast is not yet formed. b) gastrulation is complete but there are only 2 germ layers. c) the neural plate is present, but the neural tube is not yet complete d)placental development is complete, but the embryo is resistant to teratogens e) the sclerotome cells have begun migrating, but the somites have not yet formed a 2 yr old girl brought to the E.R 20 min after she accidently ingested pesticide. her father says he was spraying his lawn and accidently left the bottleopen on te ground. she is in moderate respi distress. her respi rate is 18/min and shallow. pulse oximetry on room air shows O2 saturation-- 82%. there's copious oral secretions. in addition to securing an airway, the most appropriate immediate step is administration of which of the foll medications? a--- atropine b--diphenhydramine c--physiostigmine d--pralidoxime e--succinylcholine 3) writing the qq in short--- 56 yr old woman-- 165 cm(5 ft 5 in) tall, 55kg, BMI-- 20 kg/m2. ecg--- ventricular tachycardia. doctor gives i.v bolus of antiarrythmic agent to attain an initial peak serum concentration of 10 mg/l. the vol of distribution of the drug is 1.81 l/kg. which of the foll is the most appropriate LOADING DOSE of this drug for this patient ( in mg) ? a--100 b--250 c--500 d--1000 e—1500 2. A 17-year-old girl with renal disease undergoes dialysis--waiting for transplantation. She is ofter angry and occasionally misses dialysis appointment. To improve her cooperation: A) Review criteria for remaing on transplantation list -- (wrong) B) Provide more time to talk with the physician C) Consult with mental health professional D) Antidepression therapy E) Arrange for institutional care 3. A 54-year-old man comes to the physician because of increasing abdominal pain for the past 2 months. He is jaudiced. CT -- a mass in the head of the pancreas with considerable extension to the stomach and biliary system. He is at greatest risk for developing: A) Bipolar disorder B) Major Deprssive disorder D) Personality change E) Schizophreniform disorde MI,ON ADMISSION,pulse oximetry30%O2. O2 SATURATION95% 3HR Later, SOB, O2 saturation 90%, crackles lg base, 2/6 systolic murmur, pH 7.41, PCO236,PO2 60, CAUSE OF HYPOXEMIA? A. decrease erythrocyte transit time in pul capillaries b. de''''''''''''''' lym drainage c.in'''''''''''''''''''' permeability of pul capillaries d.in''''''''''''''''''''pul capillaries pressure e.in'''''''''''''''''''''plasma colloid osmotic pressure CD44 splice correlate d/s progression, risk of relapse in hodgkin d/s[CD44 v10] archives of fresh frozen t/s ¶fin embeded biopsy screening archives method? a.affinity column chromatography b.density gradient centrifugation c.immunohistochemistry d.northern blot e.southrn blot f. RT PCR gen l/a,ct=enlage paraaortic ln, bx=prolif histiocyte neutrophil,AFB[MAI] IG A,IG G, IG M, B,T LYMPHOCYTE,CD4,CD8 [ALL NORMAL] DEFECTIVE PROTEIN? A.MHC 1 B. IFN GAMMA C.NADPH OXIDASE D. LEUCOCYTE FUN ASSO ANTIGEN 1 FEVER, BACK PAIN,103 F.PR 111, RES32,BP 115/79, L CVA TENDER, WBC 17000, PYURIA, E COLI, F D/T IL6 INDUCED BY IL1, NFKB PW FROM IL1 BIND TO IL 6 INDUCTION? A.ATTACH CYTOKINE R/C B. PHOSPHORYLATE NFKB C.RELEASE OF NFKB A/F PHOSPHORYLATION D.TRANSLOCATE TO NUCLEUS A/F PHOSPHORYLATION A 50-year-old man comes to the emergency department because of a 2-week history of progressive shortness of breath. His pulse is 90/min, respirations are 26/min, and blood pressure is 120/80 mm Hg. Physical examination shows no other abnormalities. Laboratory studies show: Arterial Pco2 30 mm Hg Arterial Po2 96 mm Hg Arterial O2 content 12 vol% (N=17%–21%) Mixed venous Po2 36 mm Hg Mixed venous O2 content 8 vol% (N=10%–16%) Which of the following is the most likely explanation for these findings? A) Anemia B) Drug-induced alveolar hypoventilation C) Residence at a high altitude D) Severe regional mismatching of alveolar ventilation and pulmonary capillary perfusion E) Voluntary hyperventilation A 22 year old man comes to the physician because of an 8 year history of episodes of yellow tinged eyes.He is medical student and he says that he has noticed that the symptoms occur during studying for final exams and other periods of intense stress.He has no pain.Physical examination shows mild scleral ictreus.Serum studies show - Bilirubin Total - 3.2mg/dl Direct - 0.4mg/dL Alkaline phosphatase - 35U/l AST -16U/l ALT - 15U/L Which of the following best explains this patients signs? A) Decreased conjugation of bilirubin B)Hepatitis C virus C)Increased hepatic copper concentration D)Intermittent obstruction of the common bile duct E)Surreptitious acetoaminophen abuse an 18 yr old woman is brought to the E.R 2 hrs after she ingested approx 100 ASA tablets in a suicide attempt. physical examination shows tachypnoea. 2 days after admission to the hospital, her hemoglobin concentration is 12 g/dl. test of the stool for occult blood is positive. which of the foll additional hematologic findings is most likely abnormal in this patient? a-- bleeding time b--fibrin degradation products c-- PTT D--platelet count e-- PT A 25 yr old woman--- comes with general malaise and facial rash for 1 week. she has a10 yr H/O episodes of pleurisy and arthritic painin peripheral joints. P/E shows an malar reash that does not involve nasolabial folds. results of cardiolipin ab, anti-dsDNA and anti-Sm ab assays are positive. which of the foll hemat abnormalities is most likely in this patient? a--HUS B--MACROCYTIC ANEMIA c--multiple nucleated RBCs d--rouleax formations e—thrombocytopenia 1. A 52-year-old man is brought to the physician 3 days after the sudden onset of blindness of the left eye. He does not have any eye pain. Funduscopy of the left eye shows a pale, opaque fundus and a bright red fovea centralis. Visual field testing shows a dense scotoma of the entire visual field of the left eye; testing of the right eye shows no abnormalities. At a follow-up examination 6 months later, the patient remains blind in the left eye. If the left eye is illuminated, which of the following reactions is most likely in the right pupil of this patient? A) Constriction because the left optic tract is binocular B) Constriction because projections to the Edinger-Westphal nucleus are bilateral C) Dilation because the posterior commissure is intact D) Dilation because the right superior cervical ganglion is intact E) No constriction because the left ciliary nerve has been permanently damaged F) No constriction because the retinal ganglion cells in the left eye have been destroyed http://img844.imageshack.us/img844/362/screenshot20130318at100.png 2. An investigator conducts a study of the molecular basis of von Hippel-Lindau (VHL) disease, specifically the role of the VHL protein in the disease process. The protein forms part of a ubiquitin- ligase complex (E3) that targets hypoxiainducible factor-1. The pathway is shown in the diagram. Step IV most likely involves which of the following organelles? A) Endoplasmic reticulum B) Golgi complex C) Lysosome D) Mitochondrion E) Proteasome 3. A paper says, "We chose the sample size to have an 80% power of detecting a 15% mean difference with a significance level (two-sided) of 5%." If there really is no difference between the groups overall, which of the following best represents the chance that the study will find a statistically significant difference, and what is this error called? A) 0%, Type I error 8) 0%, Type II error C) 5%, Type I error D) 5%, Type II error E) 15%, Type I error F) 15%, Type II error 6) 20%, Type I error H) 20%, Type II error 4. A 55-year-old woman comes to the physician because of a 3-day history of persistent rightshoulder pain. She began a weight-training program 6 weeks ago. She says that the pain intensified after she increased the amount of weight that she had been lifting above her head. She rates the pain as 8 on a 10-point scale. Examination of the right shoulder shows point tenderness just lateral to the acromion, over the humeral head. Passive motion of the shoulder is full. Pain is reproduced with resisted abduction of the shoulder when the shoulder is abducted 90 degrees and the arm is giving the "thumbs down" sign. Sensation is intact over the right upper extremity. Which of the following tendons is most likely injured in this patient? A) Deltoid B) lnfraspinatus C) Subscapularis D) Supraspinatus E) Teres major F) Teres minor http://img21.imageshack.us/img21/362/screenshot20130318at100.png 5. A 45-year-old woman comes to thephysician for a routine health maintenance examination. Several people in her family,including her son (IV,3), have hereditary telangiectasia. A pedigree is shown. Physical examination and full medical evaluation show no abnormalities. Which of the following best explains the reason for this patient's health? A) Gonadal mosaicism B) Incomplete penetrance C) Nonpaternity D) Somatic mosaicism E) Variable expressivity http://img26.imageshack.us/img26/362/screenshot20130318at100.png 6. A 72-year-old man comes to the physician because of a 3-month history of progressive burning chest discomfort after meals. Physical examination shows no abnormalities. X-rays of the chest and abdomen show a paraesophageal hernia. Based on the results of the barium swallow shown, which of the following findings is most likely present in this patient? A) Abnormal relation of the cardia to the lower end of the diaphragm B) Displacement of the pylorus inferiorly C) Gastroesophageal junction that lies two vertebral levels above the diaphragm D) Protrusion of the fundus into the chest above the level of T10 E) Right diaphragmatic hernia 7. A 52-year-old man is brought to the physician 3 days after the sudden onset of blindness of the left eye. He does not have any eye pain. Funduscopy of the left eye shows a pale, opaque fundus and a bright red fovea centralis. Visual field testing shows a dense scotoma of the entire visual field of the left eye; testing of the right eye shows no abnormalities. At a follow-up examination 6 months later, the patient remains blind in the left eye. If the left eye is illuminated, which of the following reactions is most likely in the right pupil of this patient? A) Constriction because the left optic tract is binocular B) Constriction because projections to the Edinger-Westphal nucleus are bilateral C) Dilation because the posterior commissure is intact D) Dilation because the right superior cervical ganglion is intact E) No constriction because the left ciliary nerve has been permanently damaged F) No constriction because the retinal ganglion cells in the left eye have been destroyed 8. A 1-month-old male newborn is brought to the physician for a well-child examination. He was delivered at term after an uncomplicated pregnancy, Physical examination shows exceptionally pale skin, fine white hair on the scalp, and blue eyes. His appearance was the same at birth. If a biopsy specimen of the skin were obtained in this patient, it would most likely show which of the following sets of findings? Number of Melanocytes Melanin A) Normal normal B) Normal decreased C) Decreased normal D) Decreased decreased a 51 yr old man with a 5 yr H/O wegner's comes to the physician for a follow up examination. he has general malaise, fatigue, myalgia and arthralgia. which of the foll additional findings indicate exacerbation of the pt's disease? a-- glucosuria b--hematuria c--hypokalemia d--hyponatremia e-- Incr GFR 45 YR OLD MAN-- comes for follow up 2 weeks after begining treatment with hydrochlorthiazide for HTN. he says that he feels light headed, dizzy when he gets out of bed and when he rises from a seated position too quickly. pulse-- 75/min., respi-- 12/min, B.P-- 130/85 MM HG WHEN SUPINE. no other abnormalities on P.E. which of the foll set of changes best describes changes in the CVS as this man goes from supine to standing position? -----------V.R------------CAROTID SINUS BARORECEP ACTIVITY----CEREBRAL BLOODFLOW A-------INCR-----------------INCR-------------------------------------------INCR B-------INCR-----------------INCR-------------------------------------------DECR C-------INCR-----------------DECR------------------------------------------INCR D-------INCR-----------------DECR------------------------------------------DECR E-------DECR------------------INCR-----------------------------------------INCR F-------DECR-----------------INCR------------------------------------------DECR G------DECR-----------------DECR------------------------------------------INCR H------DECR-----------------DECR------------------------------------------DECR A 30 YR OLD Nulligravid woman comes to the doctor because she has been unable to concieve for 2 yrs. she has had a 36 kg weight gain in this period. her last menstrual period was 4 months ago. menses had occured at regular 28 day intervals until the age of 25 yrs. they now occur every 90 to 120 days. she is 168 cm( 5 ft 6inch) tall and now weighs 98 kg, BMI-- 35 kg/m2. B.P- 130/90. physical and pelvic exam show no abnormalities. preg test is negative. serum studies show-- prolactin----- 15 ng/ml TSH----- 2 uU/ml FSH---- 10 MLU/ML LH---- 28 mlu/ml the PATIENT has withdrawal bleeding after a progestin challenge test. the most likely cause of this patient's oligomenorrhoea is an incr production of which of the foll hormones? a-- androgens in the adrenal gland b--- estrogen in adipose tissue c--- gonadotropin in the pituitary gland d--- gonadotropin releasing hormone in the hypothalamus e-- progesterone in the ovaries 1) a 39 year old women comes to the physician for an evaluation after moving to a new city.she has a 25 year history of multiple symptoms, including band like headaches :blurred vision,pain in her neck,back arms n legs.nausea ,irregular bowel symptoms ,urinary urgency ,difficulty attaining an orgasm,excessive menstrual bleeding.she says she has seen multiple physicians since she was a teenager ,unddergone many diagnostic tests,and has been admitted to the hospital numerous times,but no abnormalities have ever been found.she brings a copy of her medical records with her,which confirms the lack of a definitive diagnosis.which of the following is the most appropriate statement by the physicain. a) id like to asses the symptom causing u the most distress and schedule monthly follow uo appointments. b)td like to do a diagnostic work up for acute intermittent porphyria c)im sorry but i think u are malingering and most likely seeking drugs d)tell me about any emotional trauma you may have experienced during childhood e)ur symptoms could be caused by stress i would like you to undergo a psychiatric examination 2)8yr old boy is brought to the doc by his parents for a well child examination.his sister died of leukemia 4 months ago.the parents state that he is sad at times,but he has had only minor performance difficulty at school since his sister died.during the interview , the patient talks about his sisters death and cries.he then says that he has a headache.physical examination shows no abnormalities.which of the following best describes this patients condition ? a)abnormal grief reactin b)conversin disorder c)major depressive disorder d)somatoform disorder e)normal emotional response 3)a 34 yr a 34 yr old woman who is a scientist is brought to the emergency department afgter being exposed to aerosolised 125 i , while synthesisng a radiolabelled compound.she reports no symptoms other than anxiety about the exposure.she has no allergies . the most appropriate initial treatment is administration of which of the following drugs? a)levothyroxine b)liothyroxine c)methimazole d)potassium iodide E) propylthiouracil 3) 4 y/o boy with karyotype 47,xy+21 develops pancytopenia, lethargy, numerous bruises, pallor and fever. subsequent exam of bone marrow is most likely to show which: a) aplasia (wrong) b) excess lymphoblasts c) megaloblastosis d) parvovirus inclusions e) ringed sideroblasts 4) 66 y/o woman w/ ovarian ca. comes to emerg because of inability to urinate for 2 days and bilateral flank pain for 8 hrs. has not had suprapubic pain. vital signs normal. physical exam shows bilateral costovertebral angle tenderness. insertion of foley catheter yields no urine. renal ultrasonography shows bilateral hydronephrosis. which is most likely: a) decreased glom oncotic pressure b) decreased interstitial hydrostatic pressure c) decreased tubular hydrostatic pressure d) increased glomerular oncotic pressure e) increased interstitial oncotic pressure f) increased tubular hydrostatic pressure Q1) A previously healthy 18-year-old woman comes to the physician because of a 3-day history of pain with urination and urinary frequency and urgency. Physical examination shows mild tenderness over the suprapubic region. A urine culture grows an organism that is lactose fermenting and spot- indole test positive. The organism is resistant to ampicillin but sensitive to ceftriaxone. Which of the following is the most likely mechanism of antimicrobial resistance exhibited by this organism? A)Alteration of the existing penicillin-binding protein B)Changes in porin C)Efflux Pumps D)Elaboration of a new penicillin-binding protein E) b-lactamase production Q2) A newborn delivered at 28 weeks’ gestation is in severe respiratory distress. The immature alveoli of this newborn’s lungs have a diminished ability to serve as sites of effective gas exchange An increase in which of the following parameters best explains this finding? A) Alveolar radii B) Lung compliance C) Lung elastic recoil D) Pleural pressure E) Surfactant secretion Q3) A male newborn is delivered at term with a right-sided aortic arch Physical examination shows full cheeks, low-set ears, and a small chin. Serum studies show a decreased calcium concentration Which of the following is the most likely site of the malformations in this patient? A) First and second branchial arches B) First and second branchial grooves C) First and second branchial pouches D) Third and fourth branchial arches E) Third and fourth branchial grooves F) Third and fourth branchial pouches Q4) A 62-year-old man undergoes cystoscopy for investigation and removal of a suspected renal calculus. During the procedure, a cystoscope is inserted through the penis and into the bladder. Slit- like openings are then visualized near the lateral ends of the top of the trigone region of the bladder Which of the following best describes these structures? A) Ducts of the bulbourethral glands B) Ejaculatory Ducts C) Internal urethral orifices D) Ureteric orifices E) Uvulae of the bladder Q5) A 17-year-old boy with chronic renal insufficiency undergoes a unilateral nephrectomy A photograph of the resected kidney is shown Which of the following pathologic processes is most likely present in the kidney? A) Acute glomerulonephritis B) Acute tubular necrosis C) Angiomyolipoma D) ARPKD E) Fibromuscular dyspalsia F) Hydronephrosis H) Papillary necrosis I) Renal vein thrombosis J) Staghorn calculus Q6) An investigator is studying Helicobacter Pylori strains isolated from the same patient over several years. The immune response to proteins produced by these strains is observed It is found that the original Helicobacter Pylori from the patient expresses one protein recognized by the patient’s antibodies, but subsequent isolates do not express this protein Sequencing of the gene encoding the protein from the original and subsequent isolates is done It is found that subsequent isolates have nine consecutive cytidine residues, whereas the original isolate has eight The results are shown Original isolate: ACC CCC CCC ACT CAA AU GA4 CCT AGC Thr Pro Pro Thr Gln lie Glu Pro Ser Subsequent isolates: ACC CCC CCC CAC TCA AAT TGA ACC TAG Thr Pro Pro His Ser Asn STOP Which of the following mechanisms best explains this genetic change? A) Crossing over B) DNA excision repair C) Slipped-strand mispairing D) Thymidine dimer formation E) Transposon insertion Q8) A 10-month-old boy has an abdominal mass noted during a routine physical examination Abdominal ultrasonography shows a 10-cm solid mass on the upper pole of the left kidney Angiographically, the mass is hypervascular. Which of the following is the most likely diagnosis? A) Germ cell tumor B) Mesoblastic nephroma C) Wilms Tumor D) Pheochromocytoma E) RCC F) Retinoblastoma Q9) A 32-year-old man is brought to the emergency department in a coma Drug overdose is suspected Pulse is 68/mm, respirations are 8/mm and shallow, and blood pressure is 130/78 mm Hg Lung fields are clear on percussion and auscultation. Heart rate is regular with normal heart sounds Which of the following sets of arterial blood values determined while the patient is breathing room air is most likely? pH P02 Pco2 HCO3 (mm Hg) (mm Hg) (mEq/L) A) 7.15 30 30 10 B) 7.28 50 60 27 C) 7.40 80 70 42 D) 7.40 80 40 24 E) 7.50 80 30 24 Q10) A 28-year-old African American man comes to the physician because of a 3-month history of mild fatigue and weakness; he has had a 4.5-kg (1 0-Ib) weight loss during this period. He also has a 6-month history of dry cough associated with chest pain and shortness of breath. The patient is married with two children, ages 2 and 3 years. He says that no one else at home has similar symptoms. He is a carpenter. He has several pets, including a dog, two cats, and a bird. He spent a week in Mexico approximately 1 year ago. He has smoked one-half pack of cigarettes daily for 10 years. He is 178 cm (5 ft 10 in) tall and weighs 70 kg (154 lb); BMI is 22 kg/m2. Mild wheezes are heard. Physical examination shows no other abnormalities. A chest x-ray shows bilateral hilar adenopathy and right paratracheal node enlargement. Biopsy specimens obtained via fiberoptic bronchoscopy show noncaseating granulomas. Test results of the biopsy specimen for acid-fast bacilli and fungi are negative. Which of the following is the strongest predisposing risk factor for this patients condition? A) Employment B) Ethnicity C) Gender D) Living with pets E) Recent travel to Mexico F) Smoking history Q11) A 4-year-old boy with asthma is brought to the emergency department because of a 4-hour history of difficulty breathing. His symptoms do not improve with the use of an inhaled 3-adrenergic agonist Physical examination shows tachypnea and intercostal and subcostal retractions Expiratory wheezing is heard bilaterally on auscultation Following the administration of intravenous methylprednisolone, there is marked clinical improvement. Which of the following molecular pathways of methylprednisolone best explains this improvement? A) Binding to its cell surface receptor and induction of phosphorylation of JAK2 and the STAT family B) Binding to its cytoplasmic receptor, translocation to the nucleus, and activation of transcription of target genes C) Binding to its C protein-coupled membrane receptor and stimulation of adenylyl cyclase D) Binding to its serine kinase receptor on the cell membrane that signals through SMADs E) Binding to its tyrosine kinase receptor on the cell membrane that undergoes autophosphorylation and activation of multiple kinases Q12) A 14-year-old boy has had fatigue, intermittent right lower quadrant abdominal pain, diarrhea, and a 5-kg weight loss over the past 6 months A diagnosis of inflammatory bowel disease limited to the terminal ileum is made Absorption of which of the following is most likely to be impaired in this patient? A) Folic acid B) Iron C) Vitamin B1 (thiamine) D) Vitamin B2 (riboflavin) E) Vitamin B12 (cobalamin) Q13) A 75-year-old woman is brought to the physician because of a 1-day history of fever and back pain. Her temperature is 39.5°C (103.1°F), pulse is ii 1/mm, respirations are 32/mm, and blood pressure is 115/79 mm Hg Physical examination shows left-sided costovertebral angle tenderness. Laboratory studies show a leukocyte count of 17,000/mm3 (with 9% bands) and pyuria. Urine cuLtures grow Escherchia coli It is determined that the patient’s fever is partially due to interleukin-6 (IL-6), which was induced by the IL-i produced during the immune response. Which of the following best describes the role of lKB in the nuclear factor-kappa B (NF-KB) signal transduction pathway from IL-i binding to IL-6 induction in this patient? A) Attaches to cytokine receptor B) Facilitates proteolytic cleavage of NF-KB dimers C) Phosphorvlates NF-KB D) Releases NF-KB after undergoing phosphorylation F) Translocates to the nucleus after undergoing phosphorylation Q14) A 67-year-old man comes to the physician because of a 2-month history of pain in his feet. The discomfort is more severe in bed at night and is relieved by taking a hot bath. He has type 2 diabetes mellitus treated with glipizide. His pulse is 60/mm, respirations are 1 2/mm, and blood pressure is 130/88 mm Hg. Strength is normal and symmetric in the distal and proximal upper and lower extremities. The Achilles deep tendon reflexes are decreased, and quadriceps deep tendon reflexes are normal. Sensation to pinprick and vibration is decreased from just above the ankles distally. This patient is most likely to describe his pain as which of the following? A) Aching B) Burning C) Colicky D) Cramping E) Sharp F) Squeezing Q15) A previously healthy 35-year-old woman develops hypoxemia 35 minutes after ingesting a near- lethal dose of barbiturates She has not aspirated Which of the following sets of arterial blood gas values (in mm Hg) is most likely in this patient? Po2 Pco2 (A—a)02 A) 40 50 35 B) 40 60 40 C) 50 25 10 D) 50 80 10 E) 60 35 25 Q16) One-half hour after an uncomplicated vaginal delivery, a 30-year-old woman delivers the placenta but has continued heavy blood loss from the vagina. Her pulse is 120/mm, and blood pressure is 90/60 mm Hg. No cervical or perineal lacerations are noted. The uterus is soft, and contractions have stopped. Medical treatment with which of the following agents is likely to be most helpful? A) Androgen B) Estrogen C) Progesterone D) Prolactin E) Oxytocin Q17) An 8-year-old boy is brought to the physician by his parents for a well-child examination. His sister died of leukemia 4 months ago. The parents state that he is sad at times, but he has had only minor performance difficulty at school since his sister died. During the interview, the patient talks about his sister’s death and cries. He then says that he has a headache. Physical examination shows no abnormalities. Which of the following best describes this patients condition? A) Abnormal grief reaction B) Conversion disorder C) Major depressive disorder D) Somatoform disorder E) Normal emotional response a 27 yr old woman with a 1 month hx of shortness of braeth.her pulse is 90,resp 22 and bp 120/80.cardiac auscultation reveals an s3 and displaced apical impulse.a cxr shows marked cardiomegally and kerley b lines.echo shows an ef of 29%(normal>55%)which of the ff is the most likely diagnosis a)amylodosis b)hypertension c)constrictive pericarditis d)dilated cardiomyopathy e)aortic insufficiency 35) A 6-year-old boy with severe mental retardation and choreoathetosis is being evaluated for Lesch- Nyhan syndrome. Leukocytes are obtained for an assay of hypoxanthine-guanine phosphoribosyltransferase at saturating concentrations of phosphoribosylpyrophosphate and varying concentrations of guanine. Results of the data are shown. Guanine Velocity (Patient) (mM) μmoles/min 0.06 0.26 0.2 0.60 0.5 1.1 1.0 1.4 2.0 1.7 4.0 1.8 8.0 1.9 20 2.0 200 2.0 Which of the following is the approximate Michaelis constant [ Km] (in mM) for guanine in this patient? < 0.06 0.2 to 0.5 0.5 to 1.0 1.0 to 2.0 8.0 to 20 A 13-year-old boy is scheduled to receive chemotherapy for a leukemia that has the histologic features of malignent lymphocytes. This neoplasm is further typed for cell surface and intracellural markers specific for lymphocyte subsets. The neoplastic cells do not express the following markers: CD4, CD8, surface IgM, surface IgG, cytoplsmic IgM and Mu-heavy chain, cytoplasmic IgG, and Gamma-heavy chain. The leukemic cells express class I MHC molecules and show rearrangement of the T-lymphocyte receptor beta-chain gene D and J segments. Which of the following is the normal counterpart of these malignent lymphocytes? A) Activated cytolytic effector T lymphocytes in the circulation B) Mature IgM-secreting B lymphocytes in the yomph node C) Mature immunoglobulin-secreting plasma cells in the lymph node D) Pre-B lymphocyte progenitor of mature B lymphocytes in the bone marrow E) T-lymphocyte thymocytes localized to the thymic cortex. A 32-Year-old man has a3-month history of episodic epigastric cramping that awakens him at night and is sometimes relieved by eating. He smokes 1 pack of cigarattes and drinks 3 cups of coffee daily. Serologic testing shows Helicobacter pylori for which he receives antiiotic treatment. Which of the following lifestyle changes is most likely to decrease recurrence of these symptoms? A) Decrease intake of caffeine and fatty foods B) Decrease smoking C) Eliminate dietary lactose D) Eliminate whole wheat bread and cerals E) Increase dietary lactose F) Increase intake of whole wheat bread and cereals A 45 year old woman admitted for pneumococcal pneumonia and bacteremia. She is intubated and mechnically ventialed. Temp is 39.4 (102.9) pulse is 120/min, and bp is 90/50. Bilateral crackles are heard on asucultation. Lab studies show: Hemoglobin 13, HCT 40%, Leukocyte count 500, Platelet count 250,000. Production of which of the following proteins is most likely necessary for the leukocyte count to return to the reference range in the patient? A) angiogenesis Factor B) erythropoietin C) granulocyte colony stimulating factor D) interleukin 6 E) Protein c f) VWF 35 year old metal worker collapses after exposure to unknown chemocal at a factory. he is dyspneic but is non cyonotic and smells pf burnt almonds. the most appropiate next step is to administer which of the following? 1.amyl nitrate 2. erythropoitin 3. ethanol 4. hyperbaric o2 5. Physostigmin 9. A 27-year-old man comes to the physician because of a 3-hour history of nausea, cramping abdominal pain, and diarrhea. His symptoms developed 24 hours after he ate at a wedding reception; several other people who ate at the reception have similar symptoms. A culture of the stool grows Salmonella enterica. He is able to maintain oral hydration. His symptoms begin to resolve within 36 hours but persist in a milder form for several more days. Which of the following is most likely to occur if this patient is treated with antibiotics? A) Anaphylaxis as a result of antibiotic hypersensitivity B) Decreased risk for endocarditis C) Decreased risk for hemolytic uremic syndrome D) Establishment of a chronic carrier state in the spleen E) Prolonged fecal excretion of the organism A 36-Year-old man who is heterozygous for an LDL-receptor deficiency has a total serum cholesterol concentration of 330 mg/dL. After taking a drug that inhabits cholesterol synthesis, his total serum cholesterol concentration decreases to 200 mg/dL. Which of the following proteins would be upregulated as result of drug theraphy? A) Cholesterol acyltransferase B) Hepatic lipase C) LDL receptor D) Lipoprotein lipase E) Lysosomal cholesterol esterase A 69-year-old man with benign prostatic hyperplasia develops urinary retention 1 day after undergoing removal of a melanoma. After catheterization, a drug is administered that increases phosphoinositide turnover in the smooth muscle cells of the bladder fundus but has no effect on nicotinic synapses. This drug is most likely which of the following? A) Atropine B) Bethanechol C) Neostigmine D) Succinylcholine E) Trimethaphan 82 y/o. alcoholic, multiple injuries in a fall. recent hx. hip fx and multiple old and new eccymoses, abrasions, superfacial lacerations. 3cm laceration in her forhead. she has few teeth, and hemorrhagic macule around hair follicles in upper and lower extremeties. which is the most likely cause of her poor wound healing?? A) defective cross linking B) inability of fibronectin to bind to proteoglycan and cell surfaces. C) inadequate hydroxylation of collagen peptide D) impaired signal transduction of integrin. E) loss of cellular adhesions due to increase laminin protease A 22-year-old woman is found to be HIV positive after sexual contact with a partner with HIV infection. Combination therapy with lamivudine (3TC), ritonavir/lopinavir, and zidovudine (AZT) is initiated. Three months later, genomic typing shows that her HIV strain has become resistant to ritonavir/lopinavir. The cause of this resistance is most likely the acquisition of a mutation in a gene that is critical for which of the following viral processes? a. Adsorption and penetration b. Early protein synthesis c. Genome integration d. Late protein synthesis e. Nucleic acid synthesis f. Packaging and assembly g. Protein processing h. Release i. Uncoating A 47 ur old man is admitted to the hospital for treatment of a MI, on admission pusle oximetry on 30 percent O2 SHOWS AN O2 saturation greater than or equal to 95%. 3 hr later , p/t dev SOB. pulse oximerty now shows O2 sat of 90%. Crackels are heard at lung bases and grade 2/6 mur mur is heard. ABGA on 30% O2 shows pH 7.41,pco2 36mmhg, Po2 60 mmHg. Which of the following is most likely cause of p/t's hypoxiemia A. decreas alveolar Ventilation B decrease erythrocyte trasit time in pulmo capillaries C decrease lymph drainage D increase permeabilty of pulmo capillaries E increase plasma colloid osmotic pr F increase pulmo capilliary pr 3. A 2 mth old male infant is brought to emg dept by his mother bcoz of 2 D h/o of generalized tonic clonic seizure, myoclonus and hicupping. His mom says he has fed poorly and has been flooppy since birth. PE show decrease deep tendon rfx and hypotonia. a signaling defect in an inhibitory N/T is suspected. which of the following N/T receptors most likely contains the defect? A Ach Receptor B aspartic acid Receptor C Epinephrine Receptor D glutamate Receptor E glycine Receptor 4.A 3 yr old girl is brought to the physician bcoz of s1 yr h/o of short stature. she has no other majir illness. She is below 3rd percentile for ht and 10th per for wt. PE show coarse facial featiures and contractures of large jt . X rays show dysostosis multiplex. Plasma lysosoma enz analysis show increase beta hexosamidase, B glucoronidase, B galactosidase and alpha fucosidase activities. which of following mechnism is the most likelt cause of lysosomal enz finding in this p/t? A abn targeting of these enz to lysosomes B def of other lysosomal enz in the cyto C degradation of these enz within cyto D degradation of theese enz within lysosomes E storage ot thse enz witin cyto 5.A 32 ur old woman with 1* 1 cm areal of leukoplakia on cx.bx of Pap smear show invsive scc. Malignant cells fr this site will most likely drain first to which LN in this p/t? A femoral B inf mesenteric C int iliac D lumbar E superficial inguinal a couple had 2 children with osteogeneis imperfecta type 2 genetic analysis of the 42 yr old dad shows 1 in 10 of his sperm carry mutations of the type 1 procollagen genes associated with oi.which of the following is the best explanation for this finding? a. advanced paternal age b.consanguinity c, de novo mutation d.genomic imprinting e.germline mosaicism During the normal skeletal ms twitches, the ATP does not fall appreciably because: A) ATP is hydroxide only during relaxation B) ATP regenerated quickly by creatin phosphate CP C) CP hydrolyzed only by myosin D) the initial ATP is high 1)bone scan with DEXA showed decrease in bone density .what does it mean with -osteoblast activity - inc / dec -osteoclast activity - " -RANKL concentration - " blast dec, clast inc, RANKL inc 2)during a series of normal sleletal muscle twitches , the atp concentration does not fall appreciably beacause of which of the following a) atp is hydrolysed only durin relaxation b)atp is quickly regenerated from creatini phosphate c) creatine phosphate is hydrolyzes by myosin d) the initial atp conc is high 3)a 36 yr man heterozygous for an LDL receptor deficiency has total serum cholesterol 330mg/dl.after taking a drug that inhibits cholesterol synthesis, his total serum cholesterol concentration decreased to 200.which of the following proteins would be upregulated as a result of drug therapy a)cholesterol acltransferase b)hepatic lipase c)ldl receptor d)lipoprotein lipase e)lysosomal cholesterol esterase 1) A 38-year-old homeless man comes to the emergency department because of stomach cramps for 2 hours. He appears drunk and says that he began feeling ill after ingesting a homemade alcoholic beverage provided by a friend. His respiration are 33/min. Physical examination shows no other abnormalities. Fifteen minutes after arrival, he has visual disturbances. Laboratory studies show: Serum HCO3 - 5mEq/L Anion gap 40 mEq/L (N=8-16) Osmolality 370 mOsmol/kg H2O Arterial blood gas analysis on room air: pH 7.10 Pco2 18 mm Hg Po2 120 mm Hg A loading dose of an antidote is administered. Which of the following best describes the mechanism of action of the antidote in this patient? A) Binds methanol in the gastrointestinal tract and decreases its absorption B) Blocks tubular reabsorption and enhances urinary elimination of methanol C) Enhances metabolism of methanol by alcohol dehydrogenase and cytochrome P450 3A4 D) Inhibits alcohol dehydrogenase, blocking the conversion of methanol to formaldehyde E) Inhibits cytochrome P450 3A4, blocking the conversion of formaldehyde to formic acid -------- 2) Vision screening is performed on 4000 people who are older than 65 years. Cataracts are found in 200. Which of the following best describes the annual incidence of cataracts in this population? A) 0.2% B) 0.5% C) 2% D) 5% E) unable to calculate due to insufficient data ----------------- 3) A 55-year-old woman with colon cancer and a healthy 55-year-old woman are participating a study of colon cancer. malignant cells are obtained from the tumor in the affected patient, and normal colonic epithelial cells are procured from the healthy subject. After both cell types are treated with transforming growth factor-beta, the number of normal cells decreases, whereas the number of tumor cells remains unchanged. The tumor cells most likely express a mutation that inhibits which of the following physiologic processes? A) Apoptosis B) Cell cycle progression C) DNA repair D) Migration E) Necrosis ---------------------- 4) A 30-year-old nulligravid woman comes to the physician because she has been unable to conceive for 2 years. She has had a 36-kg(80-lb) weight gain during this period. Her last menstrual period was 4 months ago. menses had occurred at regular 28-day intervals until the age of 25 years. They now occur every 90 to 120 days. She is 168 cm (5 ft 6 in) tall and now weighs 98ky (215 lb); BMI is 35 kg/m2. er blood pressure is 130/90 mm Hg. Physical and pelvic examinations show no other abnormalities. A pregnancy test result is negative. Serum studies show: Prolactin 15 ng/mL Thyroid-stimulating hormone 2muU/mL Follicale-stimulating hormone 10 mlu/mL Luteinizing hormone 28 mlu/mL The patient has withdrawal breeding after a progestin challenge test. The most likely cause of this patient's oligomentorrhea is an increased production of which of the following hormones? A) Androgen in the adrenal glands B) Estrogen in adipose tissue C) Gonadotropin in the pituitary gland D) Gonadotropin-relaseing hormone in the hypothalamus E) Progesterone in the ovaries ---------------------- 5) A 27-year-old man comes to the physician because of a 3-hour history of nausea, cramping abdominal pain, and diarrhea. His symptoms developed 24 hours after he ate at a wedding reception; several other people are who ate at the reception have similar symptoms. A culture of the stool grows Salmonella enterica. He is able to maintain oral hydration. His symptoms begin to resolve within 36 hours but persist in a milder form for several more days. Which of the following is most likely to occur if this patient is treated with antibiotics? A) Anaphylaxis as a result of antibiotic hypersensitivity. B) Decreased risk for endocarditis C) Decreased risk for hemolytic uremic syndrome D) Establishment of a chronic carrier state in the spleen E) Prolonged fecal excretion of the organism 1) A 38-year-old man is admitted to the hospital after sustaining a gunshot wound to the abdomen. Borad-spectrum antibiotic therapy is initiated for complications from fecal contamination of the peritoneal cavity. he has been taking warfarin since receiving an artifical heart valve 6 years ago because of endocarditis secondary to intravenous drug use. During the next 5 weeks, the warfarin dosage required to maintain his prothrombin time progressively decreases. Which of the following best explains this finding? A) Decreased INR caused by drug-induced hepatotoxicity B) Decreased INR caused by hepatitis B infection C) Septic shock caused by an anaerobic aommensal such as Bifidobacterium species. D) Septic shock caused by a faculative anaerobe such as Escherichia coli E) Vitamin K deficiency caused by bacterial overgrowth in the small intestine F) Vitamin K deficiency caused by deplition of the normal gut flora ------------------ 2) A 13-year-old boy is scheduled to receive chemotherapy for a leukemia that has the histologic features of malignent lymphocytes. This neoplasm is further typed for cell surface and intracellural markers specific for lymphocyte subsets. The neoplastic cells do not express the following markers: CD4, CD8, surface IgM, surface IgG, cytoplsmic IgM and Mu-heavy chain, cytoplasmic IgG, and Gamma-heavy chain. The leukemic cells express class I MHC molecules and show rearrangement of the T-lymphocyte receptor beta-chain gene D and J segments. Which of the following is the normal counterpart of these malignent lymphocytes? A) Activated cytolytic effector T lymphocytes in the circulation B) Mature IgM-secreting B lymphocytes in the yomph node C) Mature immunoglobulin-secreting plasma cells in the lymph node D) Pre-B lymphocyte progenitor of mature B lymphocytes in the bone marrow E) T-lymphocyte thymocytes localized to the thymic cortex. --------------------- 3) A 1-year-old boy is brought to the physician by his parents because of a 4-week history of progressive generalized tonic-clonic seizures and a strange odor to his urine. He has a history of delayed development. he was adopted from an orphanage in Russia at the age of 6 months. Physical examination shows fair skin and blond hair. His phenylalanine hydroxylase gene is homozygous for a point mutation (GT->AT) in intron 12 of the affected gene that causes skipping of exon 12. Which of the following is the most likely explanation for exon skipping in this patient's affected gene? A) Alternative polyadenylation site B) Deletion of 5' untranslated region C) Expansion of trinucleotide repeat D) Gene duplication E) Nonhomologous recombination F) nonsense mutation G) RNA splice error H) X inactivation -------------------- 4) During a series of normal skeletal muscle twitches, the ATP concentration does not fall appreciably because of which of the following? A) ATP is hydrolyzed only during relaxation B) ATP is quickly regenerated from creatine phosphate C) Creatine phosphate is hydrolyzed by myosin D) The iinitial ATP concentration is high an 18 month old boy with SCID requires blood transfusion for sever anemia. administration of which of the following blood products is the most appropriate for this patient? A)irradiated packed RBC b)packed RBC with adenine-saline added c)washed packed RBC d)whole blood 2. A 54 year old man comes to the physician because of increasing abdominal pain for the past 2 months and a 9 kh (20 lb) weight loss over the past 3 months. He is jaundiced. A CT scan of the abdomen shows a mass in the head of the pancreas with considerable extension to the stomach and biliary system. He is at greater risk of developing which of the following conditions? A. Bipolar disorder B. major depressive disorder C. Memory impairment D. Personality change E. Schizophreniform disorder An 8 year old boy with moderate mental retardation is brought to the physician by his parent for routine exam. The parent say that their son has rently become more stubborn and has begun to have temper tantrums. These episodes usually involve fighting about food. The parent are concerned that he will hurt himself because he picks his skin and pulls his hair when he is frustrated. The father says " He spends his day looking for more things to eat" He is at the 10th percentile for height and 90th percentile for weight. which of the following is the most likely diagnosis? A. Angelman syndrome B. Down Syndrome C. Fragile X Syndrome D. Prader Willi Syndrome E. Williams Syndrome 4. An 84 year old woman with severe dementia is brought to the clinic by her daughter. the daughter tells the physician that her mother frequently refused to eat despite being provided with her favorite foods. The patient is relaxed and cooperative but is unable to provide a reliable history because of their dementia. She is 167 6 cm (5ft 6 in) tall and weighs 86kg(189.2lb) She weighted 90 kg (198 lb) 3 months ago. Her temperature is 37c (98.6 F) pulse 86/min blood pressure 130/76 mm. and respirations are 14/min. Examination shows no acute abnormalities. The daughter appears extremely worried and exhausted and is intermittently tearful during the visit. Which of the following interventions is most appropriate at this time? a. admit the patient to the hospital for replacement of a feeding tube and initiation of the enteral nutrition b. Admit the patient for parental alimentation and begin therapy with a selective serotonin reuptake inhibitor c. Discuss the stresses of the patient's care with the daughter and provide information regarding possible assistance d. Reassure the daughter that the weight loss is not a problem since her mother is still overweight e. Reassure the daughter that weight loss is normal in elderly patients. 5. A 56 year old woman comes to the doctor because of the 3 week history of progressive difficulty swallowing. When her symptoms began, she felt that bread and larger pieces of meat were getting stuck in her chest before passing through completely but now she is having difficulty swallowing all goods and some liquids. Doctor examinations shows normal oral coordination and nontender abdomen. An xray obtained after barium swallow shows an irregular mass at the junction. Infection of which of the following pathogen: A. Clonorchis B. CMV C. Entamoeba D. H Plylon E. MTB 2) A 50-year-old man comes to the physican because of a 2-week history of progressive shortness of breath while climbing stairs to his office every morning. His father had a major MI at 52. Stress echocardiography shows hypokinesis of the posterior left ventricle with increasing activity levels. Which is the most likely cause of the posterior left ventricle findings in this patient? A. Disruption of the sympathetic nerves to the LV B. Extravascular compression of the coronary arteries C. Increased LV end-diastolic pressure D. Increased myocardial oxygen consumption 3) A 70-year-old woman is found dead in her bed in a skilled nursing care facillity. She had a 12-year history of pooly controlled hypertension. A photograph of her heart is shown, it weights 650g. Which describes the changes of the heart? A. Amyloidosis B. Hypertrophy C. Infarction D. Myocarditis 4) A 30-year-old woman has had Hodgkin disease that was diagnosed 10 years ago and was treated with radiation and chemotherapy for a 6-month period. She has remained in remission for 10 years. She now feels weak and has petechiae and eccymoses. A diagnosis of AML is made. Best explaination? A. Hodgkin disease was not cured by the treatment, and surviving neoplastic cells changed to acute leukemia. B. The patient had both Hodgkin disease and Acute Leukemia at the time of initial diagnosis, but the leukemia remained dormant for several years D. The patient has inherited a predisposition to develop malignancied caused by Epstein-Barr Virus E. Therapy for Hodgkin disease causes mutations in hematopoietic stem cells, and on mutant cell progresses to leukemia A study is conducted to evaluate the effectiveness of cough medication in children. A total of 120 patients under the age of 12 years who have been diagnosed with a viral upper respiratory tract infection are randomly selected to receive dextromethorphan or no treatment. The participants were recruited from several emergency departments in an urban area Parents complete a rating scale for their children’s cough at baseline and I day after receiving dextromethorphan or nothing The resufts show a statistically significant improvement in cough scores for patients receiving dextromethorphan (p=O.04) compared with children receiving no treatment Which of the following is the most significant limitation to the internal validity of this study? A) Lack of blinding between groups B) Limited number of participants C) Marginal statistical significance D) Study location limited to only emergency departments E) Subjective nature of the survey instrument 1)2 month year old male infant with a ventricular septal defect is bought to the physician because of poor weight gain and 5 day history of rapid breathing and difficulty feeding. The diagnosis of congestive heart failure is made,and an operation to place a synthetic patch across the septal defect is planned.Compared with heart pressures prior to the operation,which of the following heart pressure are most likely 1 month after successful operation ? left ventricular pressure Right ventricular pressure Left atrial pressure A. i i NC B. i d d C. NC i d D. d NC d E. d d i NOTE: i=increase , d=decrease, NC=no change 2)3year old girl is brought to physician because of intermittent abdominal pain,constipation and irritability for 1month.She lives with her family in a50 year old house which is currently being renovated .Physical examination shows pallor.laboratory studies show normalcyti,hypo chromic anemia and plasma concentration of5-aminolevulinic acid(delta-ALA).Her blood lead concentration is 50 microg/dl(N=less than 10). A decreased activity of which of the following enzymes is most likely in this patient? A.ferrochelatase B.porphobilinogen deaminase C.protoporphyrinogen IX oxidase D.uroporphyrinogen III E.uroporphyrinogen decarboxylase 2. 32 year old in coma, drug overdose. pulse 68, RR 8/min shallow and bp 138/78 which one corresponds to pt? ph po2 pco2 Hco3- a 7.15 30 30 10 b 7.28 50 60 27 c 7.40 30 40 24 d 7.40 80 70 42 e 7.50 80 30 24 3. 45 year old on Hydrochlorothiazide for HTN, feel dizzy when he gets up. pulse 75, RR 12/min and bp 130/85. What changes u see from supine to standing position? venous return carotid sinus baroreceptor activity cerbral blood flow a up up up b up up down c up down up d up down down e down up up f down up down g down down up h down down down f is wrong 5. 72 year old with poor controlled HTN, sever abd pain for 2 hrs. pale and lethargic. p 124, rr 16, bp 95/60. S4 is present. Abd exam --> guarding, rigidity and pulsatile preumbilical mass. wat additional finding ? a ankle brachial indices that are within normal limits b AR c asymmetric radial pulse d bp reading that are higher in the left upper extremity than in the right e decreased femoral pulses 6. 40 year old woman with breast cancer. progressive loss of peripheral vision to the left in both eyes. on examination she is unable to count fingers or detect finger movements on the left. on brain MRI the metastatic tumor to which part of brain? a occipital b optic chiasm c optic nerve d parietal lobe e temporal lobe 8. Dexa scan of 65 year old woman shows decrease in bone density wat parameters are correct? Osteoblast act. osteoclast act. kB aligand( RankL ) conc. a up up up b up dpwn up c up down down d down up up e down up dpwn f down down down 3. 65 year old man with HTN volunteers to particiapte in a clinical trial of a newly developed loop diuretic. As part of the study, his acid-base/ volume status is monitored. AFter 3 days of rx, which of the following sets of findings is most likely in this patient? a)met acidosis, vol contraction b)met acidosis, no vol contraction c)met alkalosis, vol contraction d)met alkalosis, no vol contraction 10 . 66 year old woman w/ ovarian cancer. Cant urinate for 2 days. bilateral flan pain for 8 hrs. no supra pubic pain but bilateral costovertebral tenderness. foley catheter yields no urine. sono shows bilateral hydronephrosis. which one is present a dec. glo oncotic p. b dec int hydrostatic p c dec tub hydrostatic p d inc glo oncotic p e inc int oncotic p f inc tub hydrostatic p 1 24 y old woman admitted to hospital because of acute pain in left lower abdomen . she has leukocytosis and increased b-hcg, a photograph is shown [looks like a fetus in uterus]. what is the diagnosis? a..papillary ca of ovary b... ectopic pregnancy c..hadatidiform mole d.. choriocarcinoma invading myometrium e.. endometriosis 2)a 70y o man, 4month weight loss,abdominal pain and diarrhea. Stool analysis shows increased excretion of neutral fat and muscle fiber.A D-xylose test for carbohydrate absorption shows no abnormalities. Examination of tissue obtained on the intestinal biopsy shows no disorder. THis patient is most likely to respond favorably to use of which agents? A antibiotic B azathioprine C intrinsic factor D pancreatic enzyme E. prednisone 7-month-old girl is brought to the physician for a well child examination. Pulse is 110/min, respirations are 22/min, blood pressure is 95/50 mmHg. Cardiac exam shows a grade 3/6 systolic murmur that is best heard in the interscapular region. Femoral pulses are decreased bilaterally. Which of the follwing is the most likely cause of the these findings ? A) ASD B) Coaractation of the aorta C) PDA D) Total anomalous venous return E) VSD 2) An 84-year-old woman with severe dementia is brought to the clinic by her daughter. The daughter tells the physician that her mother frequently refuses to eat despite being provided with her favorite foods. The patient is relaxed and cooperative but is unable to provide a reliable history because of her dementia. She is 167.6 cm (5 ft 6 in) tall and weight 86 kg (189.2lb); BMI is 31 kg/m2. She weighed 90 kg (198 lb) 3 months ago. Her temperature is 37 C (98.6 F), pulse is 86/min, blood pressure is 130/76 mm Hg, and respirations are 14/min. Examination shows no acute abnormalities. The daughter appears extremely worried and exhausted and is intermittently tearful during the visit. Which of the following interventions is most appropriate at this time? A) Admit the patient to the hospital for placement of a feeding tube and initiation of enteral nutrition. B) Admit the patient for parenteral alimentation and begin therapy with a selective serotonin reuptake inhibitor C) Discuss the stresses of the patient's care with the daughter and provide information regarding possible assistance D) Reassure the daughter that the weight loss is not a problem since her mother is still overweight E) Reassure the daughter that weight loss is normal in elderly patients 6) 36-year-old woman is brought to the emergency department by her husband because of fever and shortness of breath for 2 hours. her husband says that she also has urinary frequency and pain with urination for 2 days. She is in respiratory distress. Her temperature is 38.7 C (101.7 F), pulse is 120/min, respiration are 30/min, and blood pressure is 80/50 mm Hg. Which of the following components of the causal organism is the most likely cause of the hypotension? A) Flagellar H antigen B) Lipopolysaccharide C) P pilus D) Polysaccharide capsule E) Shiga-like cytotoxin 1) A 23 -year-old woman comes to the physician because of a 3-day history of a drooping night eyelid. She also has a 2-week history of visual discomfort when in a room with bright lights. On examination, the right pupil is larger than the left; it also seems like the right eye is looking somewhat down and to the right. These findings most likely suggest dysfunction to which of the following structures? A) Abducens nerve B) Medial longitudinal fasciculus C) Oculomotor nerve D) Right superior cervical ganglion E) Trochlear nerve --------------- 2) An investigator is designing a study to compare a new behavioral program for attending- deficit/hyperactivity disorder (ADHD) with the standard behavioral modification program for this disorder. Because ADHD is more common among boys, girls and boys are randomized into the two treatment groups separately. Which of the following types of treatment allocation is most likely being used in this study? A) Alternation B) Open C) Outcome-adapted D) Simple random assignment E) Stratification ---------------- 3) A 23-year-old woman comes to the physician because of a 3-week history of frequent thirst and urination; she also has had a 3-kg (6.6-lb) weight loss during this period. Physical examination shows dehydration and tachypnea. Serum studies show a glucose concentration of 330 mg/dL, 2+ ketones, and pH of 7.2. Following the administration of intravenous fluids and insulin, there is marked improvement. The activity of which of the following enzymes has most likely increased in this patient's hepatocytes because of this treatment? A) Glucokinase B) Glucose 6-phoshpatase C) Glycogen phosphorylase D) Phosphoenolypyruate carboxykinase E) Phosphorylase kinase A 43 year old woman comes to the physician because of a 6 month h/o altered consciousness. During the interview, she stops talking in mid-sentence, turns to the right, extends and stiffens her right extremity. She has a blank look and doesn't respond to any question. She then has repetitive lip smacking and picking movements of the hands. The episode lasts approximately 30 seconds. She slowly returns to her normal state during the next 4-5 min. The most likely diagnosis is which of the following type of seizure? 1) Absence 2) Complex partial 3) Generalized tonic clonic 4) Simple partial A 42 year old woman with multiple sclerosis is brought to the physician because of the double vision that began yesterday morning when she awoke . The double images are horizontal on lateral gaze . Her only medication is interferon beta . On opthalmologic exam the eyes converge normally but there is a decreased adduction in both eyes qduring lateal conjugate gaze. Which is the most likely location of demyelinating plaque causing this patient's diplopia a) Lateral Lemniscus b) Medial Longitudinal fasciculus c) oculomotor never nucleus d) Rubrospinal tract e) Superior colliculus 23 year old man has a 3 month history of mildly depressed mood,decreased energy,dry skin.his thyroid gland is not palpably enlarged.serum T4 and TSH are decreased. which of the following is most likely diagnosis? 1.primary hyperthyroidism 2.secondary hyperthyroidism 3.primary hypothyroidism 4.secondary hypothroidism A previously healthy 48-year-old man is brought to the emergency department because of a 1-hour history of nausea and crushing chest pain that radiates to his left arm. An ECG and evaluation of serum cardiac enzyme activity confirm a diagnosis of a myocardial infarction of the anterior wall. Fourteen hours after arrival, he has a cardiac arrest. Resuscitation efforts are successful after approximately 30 minutes. He then immediately develops severe oliguria. Serum studies show increased urea nitrogen and creatinine concentrations. Microscopic examination of this patient's urine is most likely to show which of the following? A) Degenerating epithelial cells and dirty brown granular casts B) Degenerating WBCs and colorless hyaline casts C) Dysmorphic RBCs and yellow granular casts D) Lymphocytes and RBC casts E) Numerous eosinophils and red hyaline casts F) WBCs and RBC casts ifference in pathway of ikb of nfkb from il 1 to IL 6? a attaches to cytokine receptor b facilitates cleavage of nfkb c phosphorylates nfkb d releases nfkb after phosphoryltaion e translocation to nucleus after undergoing phosphorylation 64 YEAR OLD WITH PROGRESSIVE SHORTNESS OF BREATH WITH MINIMAL ACTIVITY FOR 2 MONTHS.BP IS 125/80.P.E. SHOWS CLUBBING.HIS TLC IS DECREASDED. A CHEST XRAY SHOWS A RECTICULAR PATTERN.WHICH OF THE FF IS THE MOST LIKELY UNDERLYING CAUSE A)CONSTRICTION OF TERMINAL BRONCHIOLES B)ALVEOLAR WALL DESTRUCTION C)INCRESED INTESTITIAL FIBROSIS D)INCRESED MUCUS SECRETION IN BRONCHOILES A 56-year-old woman comes to the physician because of a 3-week history of progressive difficulty swallowing. When her symptoms began, she felt that bread and larger pieces of meat were getting stuck in her chest before passing through completely, but now she is having difficulty swallowing all foods and some liquids. Physical examination shows normal oral coordination and a nontender abdomen. An x-ray obtained after a barium swallow shows an irregular mass at the gastroesophageal junction. A photomicrograph of a biopsy specimen obtained via esophagogastroduodenoscopy is shown. Chronic infection with which of the following pathogens is the most likely cause of the histologic findings in this patient? a)Clonorchis sinensis b)Cytomegalovirus c)Entamoeba histolytica d)Helicobacter pylori e)Mycobacterium tuberculosis 13 y old girl is brought to the doctor because of 5 month history of behavioral problems. Diagnosis of bipolar disorder is made, and treatment is started with Valproate which inhibits histone deacetylase. This drug affect which of the following processes? 1.mRNA splicing 2.Polyadenylation 3.Post-transcriptional processing 4.Transcription 5.Translation 72yr old woman comes to the physician because of diffuse muscle pain and weakness for 6 months. the muscle pain is exacerbated by activity.physical examination shows proximal muscle weakness and tenderness over the surface of both shines. the graph shows inc PTH levels and low Ca levels.which of the following is most likely cause of patients symptoms? 1.hypoparathyroidism 2.metastatic breast ca 3.osteomalacia 4.osteoporosis 5.primary hyperparathyoidism 28. An 82 years old woman with alcohol dependence is admitted to the hospital after sustaining multiple injuries in a fall. She has a recent history of right hip fracture and multiple old and new ecchymoses, abrasions and superficial lacerations. A 3 cm laceration on her forehead, sutured after a fall 6 weeks ago has dehisced. She has few teeth and there are hemorrhagic macules around her follicles on her upper and lower extremities. Which of the following is the most likely cause of her poor wound healing? a. Defective cross-linking of elastic fibers b. Inability of fibronectin to bind to proteoglycans and cells surfaces c. Inadequate hydroxylation of collagen polypeptides d. Impaired signal transduction of integrins e. Loss of cellular adhesion due to increased laminin protease a 24 yr old woman with marfan's synd comes to the physician 1 week after a home pregnancy test was positive. she has mild coarctation of the aorta with mild dilation of the asc aorta. and mild A.R, but no significant H.F. if her pregnancy was to proceed, which of the foll mechanisms wud most likely increase the amount of A.R in this patient? a--- decr vascular resistance b--hypervolemia c--incr cutaneous blood flow d--- incr metabolic rate e--weight gain a 50 yr old man comes to the E.R because of a 2 week H/O progressive shortness of breath. his pulse== 90/min, respirations--26/min and B.P--120/80. no other abn on P.E. arterial p02----------- 96 a pCO2---------------- 30 mm hg arterial O2 content--- 12 vol%( n+ 17%-21%) mixed venous pO2---- 36 mmhg mixed venous O2 content---- 8 vol%(N= 10%-16%) which of the foll is the most likely explanation for these findings? a---anemia b--drug induced alveolar hypoventilation c--residence at high altitude d--severe regional mismatching of alveolar venytilation and pulm capillary perfusion e---voluntary hyperventilation 50 yr old man--- 2 week PROGRESSIVE SOB while climbing stairs. he ha sno other probs, but he is concerned because his father had a major M.I @ 52 yrs. his pulse---110/min, regular. respi---16/min. B.P--135/95. his lungs are clear to auscultation. cardiac---- normal heart sounds with physiologic split of S2. stress echo shows hypokinesis of the post left ventricle with increasing activity levels. which of the foll is the most likley cause of the post left ventricular findings in this patient? a--- disruption of the sympathetic nerves to the left ventricle b--extravascular compression of the coronary arteries c--incr left ventricular end-diastolic pressure d---incr myocardial O2 consumption e--stenosis of the RCA 46. is lithium induced diabetes insupidus .. where there is a peripheral block of ADH , moa of adh is collecting dct so the site of block should be collecting duct . A 60-year-old woman develops a secreting adenoma of the parathyroid gland. Which of the following is most likely to be decreases? A) Calcium absorption from the gut B) Calcium concentration in feces C) Calcium concentration in muscle D) Calcium concentration in plasma E) Calcium loss from bone Man 55 yo diagnosis with wegner granulomatosis, they are asking about whats additional finding is most likely indicate worsening of the disease? A] glucosuria B] hematouria C] hypokalemia D] hyponatremia E] increase GFR 6. A 45 year oild man comes to the doctor for an initial exam. Hew has not been examed by any doctor during the last 30 years because he has not felt ill. He says that he feels well this visit. He appears relaxed. Cardiac exam showe sa grade 2/6. holsystoboliv murmur that is best heard at the left auxiliary line. Which of the following is the cause: A. AR B.AS c. MR d. MS e. TR F. TS 7. A 22 year old woman is found to be HIV positive sexual contact with ta partner with HIV infection. Combination therapy with lamivudine, ritonavir and zidovudine is initiated. 3 months latger, genormic typing shows HIV strain has become resistanct to ritonavir. the cause of this resistance is most likely acquisition of the mutation in a gene that is creicitcal of which of the viral process: A. Adsorption and penetration B. Early protein sysnthesis c. Genome integration d. late protein synthesis e. Nucleic acid synthesis f. packaing and assembly g. protein processing h. release I. uncoating
Copyright © 2024 DOKUMEN.SITE Inc.